SlideShare a Scribd company logo
1 of 72
Download to read offline
(c)

                                                                             2


                                                                            1.8


                                                                            1.6




Chapter 8                                                                   1.4


                                                                            1.2


                                                                             1




Infinite Series
                                                                            0.8



                                                                                 0     2    4        6   8        10   12
                                                                                                     x




                                                                                       n         1
                                                                   7. (a) lim             = lim          1   =1
                                                                            n→∞      n + 1 n→∞ 1 +
8.1    Sequences of Real                                                                                 n

       Numbers                                                        (b) As n gets large, n/(n + 1) gets close to 1.

      3 5 7 9 11                                                      (c)
 1. 1, , , , ,
      4 9 16 25 36
                                                                            0.9

   3 1 3 3 1 3
 2. , , , , ,
   5 2 7 8 3 10                                                             0.8



         2 1 1 1
 3. 4, 2, , , ,                                                             0.7
         3 6 30 180

     1 2 3 4 5 6                                                            0.6
 4. − , , − , , − ,
     2 3 4 5 6 7
                                                                            0.5

            1                                                                    0     2    4        6   8        10   12

 5. (a) lim 3 = 0                                                                                    x
        n→∞ n

      (b) As n gets large, n3 gets large, so 1/n3 goes                               2n + 1           1
          to 0.                                                    8. (a) lim               = lim 2 +             =2
                                                                            n→∞        n     n→∞      n
      (c)
                                                                      (b) As n gets large, (2n + 1)/n gets close to
               1                                                          2.

             0.8                                                      (c)

             0.6



             0.4



             0.2



               0
                   0   2   4        6     8    10   12
                                    x




             2
 6. (a) lim √ = 0
        n→∞   n
                               √                     2
      (b) As n gets large,         n gets large, so √ goes
                                                      n                                         1
            to 0.                                                  9. (a) In Q.(5), an =
                                                                                                n3

                                                             443
444                                                                                           CHAPTER 8. INFINITE SERIES


            1.0
                                                                                        3.0


                                                                                        2.5
           0.75

                                                                                        2.0

            0.5
                                                                                        1.5


                                                                                        1.0
           0.25

                                                                                        0.5

            0.0
                                                                                        0.0
                   2        4             6             8        10
                                                                                                   5        10         15          20


          As n gets larger and larger, the terms                                       As n gets larger and larger, the terms
                                 1
          of the sequence an = n3 approach to 0.                                       of the sequence an = 2n+1 approach 2.
                                                                                                               n
          Hence, an converges to 0.                                                    Hence, the sequence an = 2n+1 converges
                                                                                                                  n
                                                                                       to 2.
                          2
      (b) In Q.(6), an = √                                            10.
                           n
                                                                            1.0

           0.9

           0.8                                                              0.5


           0.7
                                                                            0.0
           0.6
                                                                                   0          10       20             30           40

           0.5
                                                                            −0.5

           0.4

           0.3                                                              −1.0

                  10   20            30            40       50
                                                                             As n gets larger, the terms of the sequence
          As n gets larger and larger, the terms                                      n         nπ       nπ
                                2                                            an =           sin    + cos    approach to 1
          of the sequence an = √n approach to 0.                                    n+1          2        2
                                    2                                        or -1. When n is of the form 4m or 4m + 1, the
          Hence, the sequence an = √n converges
                                                                             terms of the sequence an approach 1. When
          to 0.
                                                                             n is of the form 4m + 2 or 4m + 3, the terms
                             n                                               of the sequence an approach -1. Therefore, an
      (c) In Q.(7), an =                                                     does not convergent.
                            n+1
                                                                                                                 1
                                                                                 3n2 + 1       3+                n2        3
                                                                      11. lim       2−1
                                                                                         = lim                   1    =
                                                                             n→∞ 2n       n→∞ 2 −                          2
           0.9                                                                                                   n2
                                                                                               1
                                                                              5n3 − 1       5− 3
           0.8
                                                                      12. lim         = lim   n =5
                                                                          n→∞ 2n3 + 1  n→∞     1 2
           0.7                                                                              2+ 3
                                                                                              n
                                                                                                 1
           0.6                                                                   n2 + 1       n+ n
                                                                      13. lim           = lim      =∞
                                                                             n→∞ n + 1   n→∞ 1 + 1
                                                                                                 n
           0.5
                   5   10       15            20    25      30                             1   1
                                                                              n2 + 1         + 3
          As n gets larger and larger, the terms                      14. lim 3      = lim n n = 0
                                                                          n→∞ n + 1   n→∞     1
                                  n
          of the sequence an = n+1 approach 1.                                             1+ 3
                                    n
                                                                                              n
          Hence, the sequence an = n+1 converges                                                                               2
          to 1.                                                                                n+2               1+
                                                                      15. lim (−1)n                  = lim (−1)n               n
                                                                                                                               1
                                                                             n→∞              3n − 1  n→∞        3−            n
                         2n + 1                                                 1
      (d) In Q.(8), an =                                                     = ± , the limit does not exist; diverges
                           n                                                    3
8.1. SEQUENCES OF REAL NUMBERS                                                                    445

                                                                       n
16. Diverges. Even terms converge to 1 and odd            = lim √
    terms converge to −1.                                   n→∞     n2 + n + n
                                                                        1             1
                                                          = lim                   =
               n+2
                n            n+2                            n→∞     1 + 1/n + 1       2
17. lim (−1) 2        = lim 2
    n→∞       n +4       n→∞ n + 4
           1
             + 22                                     27. lim [ln (2n + 1) − ln n]
    = lim n n = 0                                         n→∞
      n→∞ 1 + 4                                                     2n + 1
              n2
              (−1)2 (n + 2)                               = lim ln
    Hence lim               =0                              n→∞       n
         n→∞      n2 + 4                                                  1
                                                          = lim ln 2 +        = ln 2
18. Diverges. Even terms are 1 and odd terms are
                                                            n→∞          n
    −1.                                                                                   nπ
                                                      28. When n is even number, cos           = 1 and
          x         1                                                                      2
19. lim x = lim x by l’Hopital’s Rule                              nπ 2n − 1              2n − 1
    x→∞ e      x→∞ e                                       lim cos               = lim           = 2.
              1                          n                n→∞       2 n+2            n→∞ n + 2
    and lim x = 0, so by Theorem 1.2 lim n =                                              nπ
         x→∞ e                      n→∞ e                 When n is odd number, cos            = 0 and
                                                                                           2
    0                                                               nπ 2n − 1
                                                           lim cos              = 0.
                 1    cos n      1                        n→∞        2 n+2
20. Note that − n ≤ n ≤ n and                             Therefore, the limit does not exist.
                e      e        e
          1               cos n
     lim    = 0, so lim          =0                   29. By L’Hopital Rule,
    n→∞ en         n→∞ en
    by the Squeeze Theorem.                                    n3 + 1        3n2     6n
                                                           lim     n
                                                                      = lim n = lim n
                       1      2                           n→∞    e      n→∞ e    n→∞ e
        en + 2          n + e2n
                                                                              6
21. lim         = lim e      1  =0                                    = lim n = 0.
    n→∞ e2n − 1  n→∞ 1 − 2n
                           e                                            n→∞ e

22. For n ≥ 1, en + 1 < 2en , so                      30. By L’Hopital Rule,
                                 n
      3n      1 3n     1     3                                  ln n                1
            > · n = ·              .                       lim √       = lim        n
    en + 1    2 e      2     e                            n→∞    n + 1 n→∞
                              n                                                    √1
          3           1 3                                                         2 n+1
    Since > 1, lim              = ∞;                                            √
          e      n→∞ 2    e                                                   2 n+1
              3n                                                        = lim
    so lim n       =∞                                                    n→∞      n
       n→∞ e + 1
                                                                                  1   1
                                                                        = lim 2     +   = 0.
         x2x             x            x                                  n→∞      n n2
23. lim       = lim x = lim              x
    x→∞ 3x      x→∞ 3         x→∞     3
                                                                            −1    cos n   1
                        2x            2               31. −1 ≤ cos n ≤ 1 ⇒      ≤       ≤ 2
                1                                                           n2     n2     n
    = lim                 = 0, by l’Hopital’s Rule,                          −1         1
       x→∞    3
                 x
                       3                                  for all n, and lim 2 = lim 2 = 0
                                                                        n→∞ n     n→∞ n
                   ln
              2        2                                  so by the Squeeze Theorem,
                3
                     x                                          cos n
    since lim           = ∞.                               lim        =0
                                                          n→∞ n2
          x→∞ 2
                    n
                n2                                                    1    cos πn     1           1
    Hence lim n = 0, by Theorem 1.2.                  32. Note that − 2 ≤          ≤ 2 and lim 2 =
           n→∞ 3                                                      n      n2       n      n→∞ n
                                                                    cos πn
           n!    1 2      n    1 n   n                    0, so lim        = 0 by the Squeeze Theorem.
24. Since   n
              =    · ···    ≥    · =   and                     n→∞    n2
          2      2 2      2    2 2   4
         n                    n!                                      1     1         1
     lim    = ∞, we have lim n = ∞.                   33. 0 ≤ |an | = n ≤ and lim       =0
    n→∞ 4                n→∞ 2                                       ne     n     n→∞ n
                            1
                                                          so by the Squeeze Theorem & Corollary 1.1,
                1       sin n
25. lim n sin     = lim       =1                                (−1)n
    n→∞         n n→∞ n   1                                lim        =0
                                                          n→∞ nen

26. lim   n2 + n − n                                                        1
    n→∞   √             √                                       ln x
            n2 + n − n    n2 + n + n                  34. lim 2 = lim       x = lim 1 = 0,
    = lim        √                                        x→∞ x        x→∞ 2x  x→∞ 2x2
     n→∞           n2 + n + n                             by l’Hopital’s Rule.
446                                                                         CHAPTER 8. INFINITE SERIES

                    ln n   ln n             ln n                is not a lower bound for S, there exist some
       Since (−1)n       ≤ 2 , we have lim       =                                                          ∞
                     n2     n         n→∞ n2                    aN , of for which aN < G + ε. Since {an }n=1
       0, by the Squeeze Theorem and Corollary 1.1.             is decreasing sequence, G < an < G + ε for
       an+1      n+4         n+2                                all n ≥ N . That is, G − ε ≤ an ≤ G + ε
                                                                                           ∞
 35.         =            ·                                     for all n ≥ N. Hence {an }n=1 converges to G.
         an      n+3         n+3
                                                                Therefore bounded and decreasing sequence is
          n2 + 6n + 8
       = 2            < 1 for all n, so                         convergent.
          n + 6n + 9                                                                              √
       an+1 < an for all n, so the sequence is decreas-     48. First we note that a2 = 3 + 2 3 > a1 . As-
       ing.                                                     sume that ak > ak−1 , then
                            n   n−1                                              √             √
 36. Since an+1 − an =        −                                 ak+1 = 3 + 2 ak > 3 + 2 ak−1 = ak
                           n+2 n+1                              Then we note that a1 < 3 and if we assume
                2
       =                 > 0, the sequence is increas-          that ak−1 < 3 then
          (n + 1)(n + 2)                                                       √        √
       ing.                                                     ak = 3 + 2 ak−1 < 3 + 2 · 3 = 3
                                                                We have shown that the sequence is increas-
       an+1      en+1        n       e·n
 37.         =           · n =             > 1 for all n,       ing with an upper bound, so it converges. The
        an       n+1        e       n+1                         limit of this sequence should be the solution to
                                       ∞
       so an+1 > an for all n, so {an }n=1 is increasing.       the equation
                                                                     √
                      3n+1                                      x = 3 + 2x, x2 − 2x − 3 = 0
                                                                (x + 1)(x − 3) = 0, x = −1, x = 3 Since
           an+1     (n + 3)!    3
 38. Since       =           =     ≤ 1, the se-                 the limit of the sequence can’t be negative, so
            an         3n      n+3                               lim an = 3.
                    (n + 2)!                                    n→∞
     quence is decreasing.                                  49. (a) a1000 ≈ 7.374312390,
               3n2 − 2  3n2 − 2                                     e2 ≈ 7.389056099
 39. |an | =     2+1
                       = 2                                          b1000 ≈ .135064522,
               n         n +1
           3n2     3n2                                              e−2 ≈ .135335283.
     < 2         < 2 =3                                                      m
        n +1        n                                           (b) Let n = , m = nr.
                                                                              r
                                                                                                           m/r
               6n − 1   6n + 18                                                 r n           r2
 40. |an | =          <         =6                                   lim 1 +        = lim 1 +
               n+3       n+3                                        n→∞         n    m→∞      m
                                                                                                     r
                                                                                             m/r 2
             sin(n2 )    1   1                                                       1
 41. |an | =          ≤     ≤ for n > 1.                              = lim      1+                      = er
              n+1       n+1  2                                          m→∞         m/r2
                                                                      as m/r2 → ∞
 42. |an | = e1/n ≤ e1 = e
                    n−4
 43. an = −(−2)                                             50. (a) For the first 8 terms we get:
               1
                                                                    a1 = 1,
 44. an =    2n−1                                                   a2 = 2.5,
             2n−1                                                   a3 = 2.05,
 45. an =     n2                                                    a4 = 2.000609756,
             (−1)n+1 n                                              a5 = 2.000000093,
 46. an =     (n+1)2                                                a6 = 2.000000000,
                             ∞                                      a7 = 2.000000000,
 47. Suppose that {an }n=1 is decreasing and
                                                                    a8 = 2.000000000
     bounded sequence. Then for some M > 0,
                                                                                         1       4
     |an | < M for all n. Let S be the set con-                     The equation L =         L+       has two
     taining all of the terms of the sequence, S =                                       2       L
                                                                    solutions, L = 2 and L = −2. Since the
     {a1 , a2 , a3 , ...} . By Completeness Axiom, S
                                                                    terms of the sequence are positive, we dis-
     must have a greatest lower bound say G. That
                                                                    card the negative solution. Thus the limit
     is, G is the greatest number for which an > G
                                                                    must be L = 2.
     , for all n . Therefore, for any number ε >
     0,G + ε > G and so,G + ε is not a lower bound,              (b) Assuming the limit exists, and letting L =
     since G is greatest lower bound. Since G + ε                    lim an+1 = lim an , we have
                                                                      n→∞          n→∞
8.1. SEQUENCES OF REAL NUMBERS                                                                              447

            1       c                                                   converges for p < −1 and p ≥ 1.
         L=    L+      ⇒ 2L2 = L2 + c
            2      L    √                                                              1
         ⇒ L2 = c ⇒ L = c.                                    (b) p > 0 ⇒ lim             =0
                                                                                   n→∞ np
                                                                                       1
51. Use induction to show that an+1 > an . First √                      p = 0 ⇒ lim p = 1
                                                                                n→∞ n
    note that a2 = 2, and since a2 = 2 + 2,
                  1                   2                                                1
    it follows that a2 > a2 , so a2 > a1 . Thus,                        p < 0 ⇒ lim p does not exist
                      2       1                                                 n→∞ n
    the statement is true for n = 1. Now assume                         Therefore, the sequence an = 1/np
    that the statement is true for n = k (that is,                      converges for p ≥ 0.
    ak+1 > ak ), and show that the statement is
    true for n = k + 1. First note that                               1
                    √                                   53. an =         (1 + 2 + 3 + · · · + n)
    ak+2 = 2 + ak+1                                                   n2
                  √                                                 1 n(n + 1)        n+1        1    1
    a2 = 2 + ak+1                                             = 2                  =          =    1+
      k+2
      2           √                                               n         2           2n       2    n
    ak+2 − 2 = ak+1                                                        1               1     1
                2                                              lim an =       lim 1 +          =
     a2 − 2 = ak+1
        k+2                                                   n→∞          2 n→∞          n      2
    The previous equation is also valid if we replace         Thus, the sequence an converges to 1/2.
                             2
    k by k − 1: a2 − 2 = ak
                    k+1
                                                              Note that
                                                                                   n
                                                                  1
    Since ak+1 > ak , it follows that                                                 1k
                        2                                           xdx = lim
    ak+1 > a2 − 2 and therefore
               k+1                                              0          n→∞
                                                                                  k=1
                                                                                      nn
               2               2                              Therefore the sequence an converges to
     a2 − 2 > a2 − 2
      k+2             k+1                                          1
    a2 − 2 > a2 − 2
     k+2          k+1                                                  xdx.
    a2 > a2 , ak+2 > ak+1
     k+2     k+1                                               0
    Thus, by induction, an is increasing. Now we’ll                                     n
    prove that an < 2 by induction. First note                                                 1
                                                        54.    lim an = lim
    that a1 < 2, and assume that ak < 2. Then                 n→∞                 n→∞         n+k
                                                                                        k=1
                   √                                                                     n
    ak+1 = 2 + ak                                                                             1 n
                 √               2                                            = lim
    a2 − 2 = ak , a2 − 2 = ak
     k+1                k+1
                                                                                  n→∞         nn+k
               2                   √                                                    k=1
     a2 − 2 < 2, a2 − 2 < 2
      k+1
                                                                                         n
                                                                                              1    1
                 √ 2 k+1
    a2 < 2 + 2, ak+1 < 4                                                      = lim
     k+1                                                                          n→∞         n 1 + k/n
    ak+1 < 2 Thus, by induction, an < 2.                                                k=1
                                                                                   1
    Since an is increasing and bounded above by                                1                   1
                                                                              =    dx = ln |1 + x||0
    2, an converges. To estimate the limit, we’ll
                                              √                           0  1+x
    approximate the solution of x =
              √                            2 + x:                     = ln 2 − ln 1 = ln 2
    x2 = 2 + x, (x2 − 2)2 = x                                 Therefore the sequence converges.
    x4 − 4x2 + 4 = x, 0 = x4 − 4x2 − x + 4
    0 = (x − 1)(x3 + x2 − 3x − 4)
                √                                       55. (a) Begin by joining the centers of circles C1
    Since an > 2, it follows that x = 1.                        and C2 with a line segment. The length
    Therefore, 0 = x3 + x2 − 3x − 4. Using a CAS,               of this line segment is the sum of the radii
    the solution is x ≈ 1.8312.                                 of the two circles, which is r1 + r2 . Thus,
                        1                                       the squared of the length of the line seg-
52. (a) p > 1 ⇒ lim        =0                                   ment is (r1 +r2 )2 . Now, the coordinates of
                   n→∞ pn
                        1                                       the centers of the circles are (c1 , r1 ) and
         p = 1 ⇒ lim n = 1                                      (c2 , r2 ). Using the formula for the dis-
                 n→∞ p
                            1                                   tance between two points, the square of
         0 < p < 1 ⇒ lim n does not exist                       the length of the line segment joining the
                      n→∞ p
                              1                                 two centers is (c2 −c1 )2 +(r2 −r1 )2 . Equat-
         − 1 < p < 0 ⇒ lim n does not exist                     ing the two expressions, we get
                        n→∞ p
                          1                                     (c2 − c1 )2 + (r2 − r1 )2 = (r1 + r2 )2
         p = −1 ⇒ lim n does not exist                          Expanding and simplifying this relation-
                    n→∞ p
                          1                                     ship, we get
         p < −1 ⇒ lim n = 0
                    n→∞ p                                       (c2 − c1 )2 = (r1 + r2 )2 − (r2 − r1 )2
                                                                     2             2      2              2
         Therefore, the sequence an = 1/pn                      = r1 + 2r1 r2 + r2 − (r2 − 2r1 r2 + r1 )
448                                                                    CHAPTER 8. INFINITE SERIES

          = 4r1 r2                                      57. Given that s2n = 2 −         4 − sn 2 and
                        √
          |c2 − c1 | = 2 r1 r2                              s6 = 1. Therefore,
      (b) The same reasoning applied to the other
          two pairs of centers yields analogous re-         S12 =     2−   4 − S6 2
          sults. Without going through the motions                                               √
          again, you can simply take the results                  =   2−   4 − 12 =         2−       3
          above and first replace all subscripts “2”
          by “3” to get the results for circles C1          S24 =     2−   4 − S12 2
          and C3 . Then take these new results and
                                                                                        √
          replace all subscripts “1” by “2” to get                =   2−   4− 2−            3
          the results for circles C2 and C3 . The
          results are                                                           √
          (c3 − c1 )2 + (r3 − r1 )2 = (r1 + r3 )2                 =   2−   2+       3
                        √
          |c3 − c1 | = 2 r1 r3
          (c3 − c2 )2 + (r3 − r2 )2 = (r2 + r3 )2           S48 =     2−   4 − S24 2
                        √
          |c3 − c2 | = 2 r2 r3
                                                                                                 √
                                                                  =   2−   4− 2−            2+       3
      (c) Finally,
          |c1 − c2 | = |c1 − c3 | + |c3 − c2 |
            √          √           √
          2 r1 r2 = 2 r1 r3 + 2 r2 r3                                                    √
          √          √ √           √                              =   2−   2+       2+       3 ≈ 0.130806
            r1 r2 = √ 3 ( r1 + r2 )
                       r
          √             r1 r2                              As S6 = 1, the length of each side of the regu-
            r3 = √         √
                     r1 + r2                               lar hexagon inscribed in the circle is 1 and the
      (d) Given that r1 = r2 = 1, from Part(c), we         corresponding angle subtended at the centre
                                                                             π
          have      √                                      of the circle is , which implies the radius of
          √           r1 r2      1                                           3
            r3 = √       √ =                               the circle is 1. As n increases the length of the
                   r1 + r2
                    √            2                         arc is approximately equal to the length of the
          √           r2 r3        1/2     1                                                             2π
            r4 = √       √ =             =                 side of the regular n-gon. Therefore, Sn ≈
                   r2 + r3
                    √            1 + 1/2   3                                                              n
                                                                            π
          √           r3 r4         1/6       1            . Thus, S48 ≈       .
            r5 = √       √ =               =                                24
                   r3 + r4       1/2 + 1/3    5
                            1        1         1
          The pattern is √ = √           +√
                            rn      rn−1      rn−2
          Hence if Fn is the n-th Fibonacci number,
                          2
          then rn = 1/Fn .

 56. (a) The distance between the two points
         (0, c) and (x0 , y0 ), where y0 = x2 , is r,
                                            0

             x2 + (x2 − c)2 = r
              0     0
          x2 + x4 − 2cx2 + c2 = r2
           0     0      0
          y0 + (1 − 2c)y0 + (c2 − r2 ) = 0
           2                                                     n!     1·2·3·4···n
                                                        58. 0 <   n
                                                                     =
          We want the solution y0 to the above                  n       n·n·n·n···n
          equation to be unique, so that                         1 2·3·4···n          1                  1
                                                            =                     <              (1) =
          (1 − 2c)2 − 4(c2 − r2 ) = 0                            n n·n·n···n          n                  n
          1 − 4c + 4c2 − 4c2 + 4r2 = 0                                   n!    1
                                  1                        Hence, 0 < n < .
          1 − 4c + 4r2 = 0, c = + r2                                    n      n
                                  4                        Therefore,
                                                                            n!       1
      (b) Following Part(a), the pattern is                 lim 0 < lim n < lim
                                                           n→∞        n→∞ n      n→∞ n
           2   1                   1       2
          rn + = rn + rn−1 + + rn−1                                    n!
               4                   4                       0 < lim n < 0
           2    2                                               n→∞ n
          rn − rn−1 = rn + rn−1
          rn − rn−1 = 1                                           n!
                                                            lim       =0
          r1 = 1, r2 = 2, r3 = 3, . . . rn = n             n→∞ nn
8.2. INFINITE SERIES                                                                                              449

       2         0.5                                 61. In the 3rd month, only the adult rabbits have
       3       0.5436                                    newborns, so a3 = 2+1 = 3. In the 4th month,
       4       0.5731                                    only the 2 pairs of adult rabbits from a2 can
       5       0.5949                                    have newborns, so a4 = 3 + 2 = 5. In general,
       6       0.6119                                    an = an−1 + an−2
       7       0.6258
       8       0.6374                                62. Let an and an+1 denote the sizes of the shorter
       9       0.6473                                    and longer sides of the nth rectangle. For ex-
       10      0.6559                                    ample, a1 = 1 and a2 = 2 are the sides of the
                                                         first rectangle, a2 = 2 and a3 = 3 are the sides
       20      0.7065
                                                         of the second rectangle, and so on. When a new
       30      0.7316
                                                         rectangle an ×an+1 is formed, the longer side of
       40      0.7476
                                                         the previous one, an , becomes the shorter side
       50      0.7590
                                                         of the new one, and the longer one of the new
       60      0.7678
                                                         one, an+1 , is the sum of the shorter one and the
       70      0.7748                                    longer one of the previous one. If we express
       80      0.7806                                    that as a formula we get an+1 = an−1 + an ,
       90      0.7855                                    which is the property defining the Fibonacci
      100      0.7898                                    sequence.
      200      0.8146
      300      0.8269
      500      0.8403                                8.2       Infinite Series
     1000      0.8589                                      ∞                 k
                          ln (n!)                                    1
    From the table, lim           = 1.                1.         3               is a geometric series with
                    n→∞ ln (nn )                                     5
                                                           k=0
    When x → ∞, slope of the graph y = ln x ap-                            1
    proaches to 0. Due to this property, the given         a = 3 and |r| =   < 1, so it converges to
                                                                           5
    limit approaches to 1.                                    3      15
                                                                   =
                                                           1 − 1/5    4
                                          12               ∞
59. If side s = 12 and the diameter D =      then               1 k
                                          n           2.          5 is a geometric series with
                                                                3
    the number of disks that fit along one side is          k=0
      12                                                   |r| = 5 > 1, so it diverges.
      12   = n. Thus, the total number of disks is
      n                                                    ∞                     k
     12        12                                                1       1
           ·        =n·n=n   2                        3.             −               is a geometric series with
      12       12                                                2       3
      n        n                                           k=0
    an = wasted area inbox with n2 disks                        1            1
                       2                                   a =     and |r| =   < 1, so it converges to
                   6                                            2            3
    = 12 · 12 − n2       π = 144 − 36π ≈ 30.9                  1/2       3
                   n                                                  = .
                                                           1 − (−1/3)    8
                                                           ∞                 k
60. The answer is a5 = 30 or a5 = 31, depending                      1
    on the position of the points.                    4.         4               is a geometric series with
                                                                     2
                                                           k=0
                                                                           1
                                                           a = 4 and |r| =   < 1, so it converges to
                                                                           2
                                                              4
                                                                   = 8.
                                                           1 − 1/2
                                                           ∞
                                                                1 k
                                                      5.          (3) is a geometric series with
                                                                2
                                                           k=0
                                                           |r| = 3 > 1, so it diverges.
                                                           ∞                     k
                                                                         1
                                                      6.         3 −                 is a geometric series with
                                                                         2
                                                           k=0
450                                                                                       CHAPTER 8. INFINITE SERIES

                       1                                         13. Using partial fractions
       a = 3 and |r| =   < 1 so it converges to                             n
                       2                                                         2k + 1
            3                                                        Sn =
                  = 2.                                                        k 2 (k + 1)2
       1 − (−1/2)                                                                  k=1
                                                                                    n
  7. Using partial fractions,                                                             1       1
                                                                             =               −
            n
                    4
                              n
                                  2     2                                                 k2   (k + 1)2
                                                                                   k=1
     Sn =                =          −
                k(k + 2)          k k+2                                                    1       1 1           1   1
           k=1              k=1                                              =      1−         +    −        +     −
              2            2       2 2                                                     4       4 9           9 16
     = 2−         + 1−          +    −
              3            4       3 5                                                     1      1
                  2      2                                                      + ··· +       2
                                                                                                − 2
        + ··· +      −                                                                 (n − 1)   n
                  n n+2                                                          1         1
                   2         2                                                +     −
     =2+1−             −                                                         n2    (n + 1)2
                 n+1 n+2
               4n + 6                                                                1
     =3− 2               and                                                 =1−
            n + 3n + 2                                                            (n + 1)2
                                4n + 6                                                                1
      lim Sn = lim 3 − 2                  = 3.                         and, lim Sn = lim             1−               =1
     n→∞         n→∞          n + 3n + 2                                        n→∞            n→∞(n + 1)2
     Thus, the series converges to 3.                                  Thus the series converges to 1.
              4k                                                       ∞
  8. lim         = 4 = 0, so the series diverges by                                      4
       k→∞ k + 2                                                 14.
            th
       the k -Term Test for Divergence.                                        k(k + 1)(k + 3)(k + 4)
                                                                       k=1
                                                                                ∞
            3k                                                             4                1           1
  9. lim        = 3 = 0, so by the k th -Term Test                     =                         −
     k→∞ k + 4
                                                                           3             k(k + 4) (k + 1)(k + 3)
                                                                               k=1
     for Divergence, the series diverges.                                4
                                                                                ∞
                                                                                        1      4
                                                                                                     ∞
                                                                                                                 1
                                                                       =                     −
                                                                         3           k(k + 4) 3            (k + 1)(k + 3)
 10. This is a telescoping sum:                                                k=1
                                                                                 ∞
                                                                                                     k=1
            n                  n                                         41                1   1
                   9                3      3                           =                     −
     Sn =                 =           −                                  34                k k+4
               k(k + 3)             k k+3                                          k=1
           k=1               k=1
                                                                                      ∞
       3 3 3 3                     3      3                                  41                 1   1
     = − + − + ··· + −                                                     −                      −
       1 4 2 5                     n n+3                                     32                k+1 k+3
            3 3         3          3         3                                       k=1
     =3+ + −                 −          −      .                                ∞
            2 3 n+1 n+2 n+3                                              1               1   1
            ∞                                                          =                   −
                   9                                                     3               k k+4
                                                                               k=1
     Thus                                                                         ∞
               k(k + 3)                                                        1         1   1
           k=1                                                             −               −
                    3 3          3        3                                    6        k+1 k+3
     = lim 3 + + −                    −                                            k=1
       n→∞          2 3 n+1 n+2                                         1              1 1 1   1              1 1
            3                                                          =           1+ + +    −                 +
       −                                                                3              2 3 4   6              2 3
         n+3                                                            5
              3 3         11                                           = .
     = 3+ +           =       .                                         9
              2 3          2
       ∞            ∞                                  ∞               ∞
             2            1                                  1             2
 11.           =2           and from Example 2.7,                15.            is a geometric series with
             k            k                                  k             ek
       k=1          k=1                                k=1             k=2
                          ∞                                                 2              1
                                1                                      a = 2 and |r| = < 1,so it converges to
       diverges, so 2             diverges.                                 e              e
                                k                                             2            2
                          k=1                                                          = 2      .
                                                                       e 2 1− 1          e −e
                                                                                  e
       ∞                   ∞
              4                  1
 12.             = 4               and the harmonic series
             k+1                 k                               16. lim |ak | = lim 31/k = 30 = 1 = 0 , So by
       k=0                 k=1
        ∞                        ∞                                     k→∞                 k→∞
             1                          4                              the k th Term Test for Divergence, the series
               diverges, so                diverges.
             k                         k+1                             diverges.
       k=1                       k=0
8.2. INFINITE SERIES                                                                                           451

      ∞                        ∞            ∞
             1       1             1          1                 gent if |r| = |2c + 1| < 1 .
17.             −          =          −          . The
             2k    k+1             2k        k+1                That is, if −1 < 2c + 1 < 1 or −2 < 2c < 0
      k=0                     k=0       k=0
      first series is a convergent geometric series but          Therefore, the series converges for
      the second series is the divergent harmonic se-           −1 < c < 0.
      ries, so the original series diverges.
      ∞                    ∞            ∞
         1    1        1         1                              ∞
18.         − k =          −                                             2
         2k  3         2k        3k                       26.                k
                                                                                  is a geometric series with a = 2
    k=0            k=0       k=0                                      (c − 3)
         1       1       1                                      k=0
    =        −        =                                                             1
      1 − 1/2 1 − 1/3    2                                      and |r| =                . The series converges, if
                                                                                 (c − 3)
      ∞                    ∞            ∞                                 1
            2     1           2        1                        |r| =          < 1, that is |c − 3| > 1, there-
19.            + k =             +        .                             c−3
            3k   2            3k       2k
      k=0                 k=0      k=0                          fore c > 4, c < 2. Thus the series converges for
      The first series is a geometric series with
                           1                                    c < 2 and c > 4.
      a = 2 and |r| =         < 1 so it converges to
                           3
          2
               = 3.                                             ∞
      1 − 1/3                                                        c
      The second series is a geometric series with        27.             is divergent for all values of c. Refer
                           1                                        k+1
                                                                k=0
      a = 1 and |r| =        < 1, so it converges to            Excercies 12
                           2
          1
               = 2.
      1 − 1/2
             ∞
                  2      1                                28. Two cases possible.
      Thus           + k = 3 + 2 = 5.
                 3k     2                                     (i) c = 0: ⇒ given series is
              k=0
                                                               ∞              ∞
      ∞                   ∞         ∞                                  2
             1    1           1         1                                   =    2 which is divergent
20.            − k =             −                                 (ck + 1)
             k 4              k         4k                      k=0                   k=0
      k=0                 k=0      k=0
      The second series is a convergent geometric se-
      ries, but the first series is a divergent harmonic         (ii) c = 0: ⇒ given series is
                                                                 ∞                ∞
      series, so the series diverges.                                     2             2
                                                                               =
                                                                      (ck + 1)       c k+ 1 c
                        3k                                      k=0              k=0
21. lim |ak | = lim          =3=0                                                         ∞
      k→∞          k→∞ k + 1                                                          2          1
                    th
      So by the k -Term Test for Divergence, the                                  =                   1
                                                                                      c         k+    c
                                                                                          k=0
      series diverges.                                                                          ∞
                                                                                                       1
                            k   3                               Which is divergent as                      1
22. The limit lim (−1)k 2         does not exists, so                                           k=0
                                                                                                      k+   c
               k→∞        k +1                                  is divergent.
                                th
    the series diverges by the k -Term Test for Di-
    vergence.
23. Since k is positive integer, k is ration num-
                                 5                        29. The series appears to converge.
    ber and can’t be multiple of π.         Hence                         n  1
              k                                                  n          √
     lim sin      = 0. Therefore , the series di-                       k=1   k
    k→∞       5
    verges.                                                      2    1.250000000
                                                                 4    1.423611111
                                 π                               8    1.527422052
24. The limit lim tan−1 (k) =       = 0, so by the              16    1.584346533
                    k→∞          2
      k th Term Test for Divergence, the series di-             32    1.614167263
      verges.                                                   64    1.629430501
      ∞                                                         128 1.637152005
                     k                                          256 1.641035436
25.         3(2c + 1) is a geometric series with
      k=0                                                       512 1.642982848
      a = 3 and |r| = |2c + 1| . The series is conver-         1024 1.643957981
452                                                                                       CHAPTER 8. INFINITE SERIES



                                                                        5
       1.6


       1.5                                                           4.5



       1.4                                                              4



       1.3
                                                                     3.5


       1.2
                                                                        3
                                                                            0         20         40      60         80    100
       1.1


            1                                                  32. The series appears to converge.
                0         20        40        60    80   100

                                                                                      n
                                                                                           2k
                                                                    n
                                                                                           k
                                                                                     k=1
 30. The series appears to diverge.
                 n  1                                                1               2
        n          √                                                 2               4
               k=1   k
                                                                     3          5.333333333
        2    1.707106781
                                                                     4               6
        4    2.784457050
                                                                     5          6.266666667
        8    4.371436800
                                                                     6          6.355555556
        16 6.663994609
                                                                     7          6.380952381
        32 9.941512173
                                                                     8          6.387301587
        64 14.60206410
                                                                     9          6.388712522
       128 21.21122790
                                                                    10          6.388994709
       256 30.57088534
       512 43.81657304
      1024 62.55526935                                                6




                                                                      5
       12


       10
                                                                      4


        8
                                                                      3

        6


                                                                      2
        4
                                                                                      10         20      30         40     50


        2
                                                                                                ∞
                     10        20        30   40   50          33. (a) Assume                        ak converges to L.
                                                                                               k=1
                                                                                Then for any m,
                                                                                          ∞           m−1            ∞
 31. The series appears to converge.                                            L=             ak =          ak +         ak
                                                                                      k=1              k=1          k=m
                          n                                                                            ∞
                           1
        n                 √                                                          = Sm−1 +               ak .
                      k=1   k                                                                         k=m
        2           4.500000000                                                       ∞

        4           5.125000000                                                 So            ak = L − Sm−1 ,
                                                                                     k=m
        8           5.154836310
                                                                                and thus converges.
       16           5.154845485                                                            ∞                                    ∞
       32           5.154845485                                     (b) Since                   ak = (a1 + · · · + am−1 ) +           ak
      1024          5.154845485                                                           k=1                                   k=m
8.2. INFINITE SERIES                                                                                                                    453

                                                                                                     ∞1
           and the finite sum a1 +· · ·+am−1 does not                              37. Let Sn =          . Then
           affect convergence, both series converge or                                           k=1 k
           both diverge.                                                                                      1    3         5
                                                                                      S1 = 1 and S2 = 1 + = . Since S8 > , we
                                                                                                              2    2         2
34. By using partial fraction technique, we get:                                      have
                                                                                                   1      1           1
         1           1       1                                                        S16 = S8 + +          + ··· +
             1 =2       −      1                                                                   9 10              16
    k k+ 2           k     k+ 2                                                                   1             1      5 1
    It may be observed that unlike example 2.3                                        > S8 + 8           = S8 + > + = 3.
                                                                                                  16            2      2 2
           1        1                                                                  So S16 > 3.
    both and              do not form the same se-
           k     k+ 1                                                                                1      1            1
                       2                                                              S32 = S16 +        +    + ··· +
    ries. Consequently, the terms in the nth par-                                                   17 18               32
    tial sum do not get canceled. Hence the said                                                      1             1      1 7
                                                                                      > S16 + 16           = S16 + > 3 + =
    method is not useful for this example.                                                           32             2      2 2
                                                                                                7
                 ∞                                              ∞                     So S32 > .
                                                                                                2
35. Assume             ak converges to A and                         bk con-          If n = 64, then S64 > 4. If n = 256, then
                 k=1                                           k=1                    S256 > 5. If n = 4m−1 , then Sn > m.
    verges to B. Then the sequences of partial
    sums converge, and letting                                                    38. The nth partial sum of the series
            n                            n
    Sn =         ak and Tn =                   bk ,                                   P = 1 − 1 + 1 − 1 + 1 · ·· is given by
                                                                                            n
           k=1                           k=1                                                        k   1             n
    we have lim Sn = A and lim Tn = B.                                                Pn =     (−1) = (1 + (−1) ).
                  n                            n
                                                                                                        2
                                                                                               k=0
                      n
                                                                                      Here Sn = the nth partial sum of the series
    Let Qn =              (ak + bk ), the sequence of partial
                                                                                      1 − 1 + 1 − 1 + 1 · ··
                  k=1
                  ∞
                                                                                                             1     n
                                                                                      Which is =Sn = 1+ (1 + (−1) ).
    sums for              (ak + bk ). Since S, T , and Q are                                                 2
                                                                                           n
                 k=1                                                                                   k
                                                                                      As         (−1) diverges, by theorem 2.3 the se-
    all finite sums,
                                                                                           k=0
    Qn = Sn + Tn .                                                                    ries 1 + 1 − 1 + 1 − 1 + 1 · ·· diverges.
    Then by Theorem 1.1(i),
    A + B = lim Sn + lim Tn = lim(Sn + Tn )                                           For the series A = 1 + 1 − 1 + 1 − 1 + 1 · ··, nth
                  n                n               n                                                                          n−1
                                   ∞                                                                                                    k
                                                                                      partial sum is given by, Sn = 1 +             (−1) .
             = lim Qn =                  (ak + bk )                                                                           k=0
                  n
                               ∞
                                   k=1
                                                               ∞
                                                                                      Now, for the series
    The proofs for                 (ak − bk ) and                   cak are           P = 1 − 1 + 1 − 1 + 1 · ·· we can write:
                                                                                      P =1−1+1−1+1······
                             k=1                              k=1
    similar.                                                                           1 − P = 1 − (1 − 1 + 1 − 1 + 1 · · · · · ·) = P
                           ∞                                     ∞                             1
36. Assume that                  ak converges and                     bk di-            ⇒P =
                                                                                               2
                           k=1                                  k=1
                                                    ∞
                                                                                      For the 1 + 1 − 1 + 1 − 1 + 1 · · · · · · we have:
    verges. Now suppose that                             (ak + bk ) con-              A = 1 + P = 1 − 1 + 1 − 1 + 1 · ··
                                                   k=1
    verges. But, since                                                                         1    3
     ∞            ∞                        ∞                                             =1+ =
                                                                                               2    2
          bk =         (ak + bk ) −                ak                                 The Cesaro sum
                                                                                                 n
    k=1          k=1                      k=1                                                1
                                                                      ∞                lim         Sk
                                                                                      n→∞    n
    Theorem 2.3(i) implies convergence of                                  bk .                      k=1
                                                                                                            n
                                                                     k=1                    1                                3
                                                        ∞                             = lim   · lim              Sk   =0·      =0
                                                                                       n→∞ n n→∞                             2
    This contradiction shows that                           (ak +bk ) can-                                 k=1
                                                        k=1
                                                         ∞                                                            ∞
    not converge. The proof for                               (ak − bk ) is       39. (a) .9 + .09 + .009 + · · · =         .9(.1)k
                                                        k=1                                                           k=0
    almost identical.                                                                       which is a geometric series with a = .9
454                                                                      CHAPTER 8. INFINITE SERIES

           and |r| = .1 < 1 so it converges to                This can’t be an integer since all the terms on
             .9                                               the numerator are divisible by p except for the
                  = 1.
           1 − .1                                             last term.
                         1    9       9
       (b) 0.199999 =      +      +       + ...           46. Step 0: [0, 1]
                        10 100 1000                           Step 1: After removing middle third, set be-
                              ∞              k
                         1         9    1                               1     2
                    =      +                                  come 0, 3 ∪ 3 , 1
                        10       100 10                       Step 2: After removing middle third from
                             k=0
                                 9                                                            1
                                                              each subinterval, set become 0, 9 ∪ 2 , 3 ∪
                                                                                                   9 9
                         1                 2                   6 7      8
                    =      + 100       =                       9 , 9 ∪ 9, 1
                        10 1 − 1/10       10
                         18       18                          All the end points of the subintervals belong
 40. (a) 0.181818 =          +          + ...                 to Cantor Set. Therefore other four points are
                        100 10000
                            ∞                                 1 2 7 8                                        1
                                  1                           9 , 9 , 9 , 9 . Third term of the series is 4 27 .
                     = 18             .                       Sum of the series:
                                100k
                           k=1                                       1           1          1
           This is a geometric series with                     =          +2         +4          + ...
                 18                  1                               3           9         27
           a =        and |r| =         , so the sum is              1           1            1
                 100                100                        =          +2          + 22          + ...
             18/100        2                                         3           32           33
                       =     .
           1 − 1/100      11                                         1              1            1
                              134        134                   =             1+2        + 22          + ...
       (b) 2.134134 = 2 +           +           + ...                3              3            32
                             1000 1000000                          1       1
                                  ∞
                                         1                     =              2
                     = 2 + 134               .                     3 1− 3
                                      1000k
                                 k=1
                                                               =1
           The second term is a geometric series with
                134                 1
           a=         and |r| =         , so the sum is       The length of the Cantor Set
                1000              1000                         = 1 − (Sum of the lengths of remove intervals)
                134/1000         134
           2+                =2       .                        = 1 − (Sum of the series)
               1 − 1/1000        999
       ∞             ∞
                                                               =1−1
             1             −1                                  =0
 41.           and
             k             k
       k=1           k=1
                                                          47. We have from the proof of the Theorem 2.1,
 42. Yes.                                                                                    1 1 − xn+1
                                                              1+x+x2 +x3 +x4 +........+xn =              .
                                                                                                1−x
 43. The geometric series 1 + r + r2 + r3 + · · · con-        Now consider
                     1                                        0 < x < 1 ⇒ 0 < xn < 1
     verges to S =       provided that
                   1−r                                         − 1 < −xn < 0 or 0 < 1 − xn < 1
     −1 < r < 1. But −1 < r implies that −r < 1,
                                          1         1                       1 − xn       1
     and so 1 − r < 2, and therefore             > ,          therefore 0 <        <
                                        1−r         2                        1−x       1−x
                            1                                                                 1
     which means that S > .                                    ⇒ 0 < 1 + x + x2 + ... + xn <
                            2                                                                1−x
 44. Since       ak converges, lim ak = 0,                    Now, for the interval −1 < x < 0
                                k→∞
                                           1                  0 < −x < 1
       by Theorem 2.2. Therefore, lim        does not         1<1−x<2
                                     k→∞ ak
                                  1                           If n is even, −1 < xn+1 < 0. Then
       exist. In particular, lim     = 0.                     0 < −xn+1 < 1
                             k→∞ ak
                       1                                      ⇒ 1 < 1 − xn+1 < 2
       Therefore,         diverges, by the converse of              1      1 − xn+1     2
                       ak                                     ⇒         <           <
       Theorem 2.2.                                               1−x        1−x      1−x
                                                                                                   1 − xn+1
 45. Suppose p is a prime,                                    Therefore, 1 + x + x2 + ... + xn =            >
                                                                                                     1−x
        1 1          1    p! + p!/2 + · + p!/p                 1
     1+ + +·+ =
        2 3          p             p!                         1−x
8.2. INFINITE SERIES                                                                                                           455

    Thus, the inequality does not hold if n is even.             uct on the left side of the inequality. This
                                                                 product will yield the sum of the terms of the
                                                                        1
                  ∞                                              form ,where k is either a prime or a product
                     1                                                  k
48. The series          is a geometric series with               of the primes or the product of a prime and a
                    2k                                           composite or it is the product of two compos-
               k=0
                   1                                             ites, so that we can write the left side of the
    a = 1 and r = , its sum is 2.
                   2                                                                1 1 1               1
               ∞                                                 inequality as 1 + + + + · · · · + ,where
                   1                                                                2 3 4               n
    Therefore,         = 2.                                      n is a positive integer greater than the largest
                   2k
                 k=0                                             prime p.
    Now, the nth partial sum of this series is given             Thus, we have
    by                                                               1 1 1            1       3 5            p
     n
        1              1
                           n                                     1+ + + +····+ < 2· · ··········
           =2 1−               < 2                                   2 3 4            n       2 4           p−1
        2k             2                                         It may be observed that, if there would have
    k=0
                   1             1                               been finite number of primes then there would
    Therefore 1 + + ........ + n < 2            ...(i)           have been finite number of terms on either side
                   2            2
                ∞                                                of the inequality, which is not the case hence
                     1
    The series          is a geometric series with               we conclude that there are infinite number of
                    3k
               k=0                                               primes.
                   1              3
    a = 1 and r = , its sum is .                                                                         n−1
                   3              2                                                                                  L
               ∞                                           49. The amount of overhang is                                   .
                   1      3                                                                                       2(n − k)
    Therefore,         =                                                                                 k=0
                   3k     2                                                                 7
                 k=0                                                                                L
                 th                                              So if n = 8, then                        = 1.3589L.
    Now, the n partial sum of this series is given
                                                                                                 2(8 − k)
    by                                                                                  k=0
     n
        1    3           1
                             n
                                    3                            When n = 4,
           =      1−           <                                   n−1                           3
        3k   2           3          2                                     1                               1
    k=0                                                          L              =L
                    1             1      3                             2(n − k)                       2(4 − k)
                                                                      k=0                       k=0
    Therefore, 1 + + ........ + n <           ...(ii)
                    3            3       2                                           = 1.0417L > L
                 ∞
                      1                                                   n−1                            n
    The series           is a geometric series with                                L                             L
                     5k                                           lim                   = lim
                k=0                                              n→∞            2(n − k) n→∞                     2k
                    1             5                                       k=0                           k=1
    a = 1 and r = , its sum is .                                                                              n
                    5             4                                                           L                       1
                ∞                                                                           =    lim                    =∞
                    1      5                                                                  2 n→∞                   k
    Therefore,          =                                                                                    k=1
                    5k     4
                  k=0                                             n                     n
                 th
    Now the n partial sum of this series is given                        1            1       1
                                                           50.                =           −
    by                                                               k(k − 1)       k−1 k
                                                                 k=2            k=2
     n                         n
          1      4           1       5                              1 1 1 1                 1       1
             =       1−           <                              = − + − + ··· +                −
         5k      5           5       4                              1 2 2 3              n−1 n
    k=0                                                                 1
                         1            1     5                    = 1 − , we have
    Therefore, 1 + + ........ + n <             ...(iii)                n
                         5           5      4                     ∞
    In general we can write for a largest prime p,                       1                1
                                                                              = lim 1 −        = 1.
         1                1       p                                  k(k − 1) n→∞        n
    1 + + ........ + n <                                         k=2
         p               p       p−1                             On the other hand,
    Now,           multiplying     the    inequalities            ∞               ∞
                                                                           1           1
    (i),(ii),(iii).etc. We get,                                      k          =
                                                                       k(k − 1)     (k − 1)
             1              1        1         1                 k=2                    k=2
       1+ +····+ n               1+ +····+ n                                             ∞
             2             2         3        3                                                  1
                                                                                    =              = ∞.
             1              1           1           1                                            k
       1+ +····+ n ··· 1+ +····+ n                                                      k=1
             5             5            p          p                                                         2
               3 5                  p                      51. p2 + 2p(1 − p)p2 + [2p(1 − p)] p2 + · · ·
                                                                      ∞
     < 2·         ·    ··········                                                           k
               2 4                p−1                            =          p2 [2p(1 − p)] is a geometric series with
    for any positive integer n. Consider the prod-                    k=0
456                                                                              CHAPTER 8. INFINITE SERIES

       a = p2 and |r| = 2p(1−p) < 1 because 2p(1−p)                   The sum represents the probability that you
       is a probability and therefore must be between                 eventually win a game.
       0 and 1. So the series converges to
              p2               p2
                        =              .
       1 − [2p(1 − p)|    1 − 2p + 2p2
       If, p = .6,                                                                 2v
                                                                           ∞
                .62                                                   2v            g        2v
       1 − 2(.6) + 2(.6)2
                          = .692 > .6.                            58.        rk =       =
                                                                       g          1−r     g(1 − r)
                                                                         k=0
              1        p2                                                            2v
       If p > ,                > p.                                       ∞
              2 1 − 2p(1 − p)                                         2v              g         2v
                                                                             r2k =        =
                  1      1                1
                                                   ∞          k        g           1 − r2   g(1 − r2 )
                                                                           k=0
 52. Since 1 +      + 2 + ··· =
                 12 12                   12
                                    k=0
            1       12
       =         =     , the minute hand and the hour
              1     11
         1−                                                       59. (a) Take a number less than 1000 and try.
              12
       hand are in the same location approximately 5
       minutes and 27 seconds after 1.
                                                                       (b) The series
                                               ∞                                 c      c      c
 53. d + de−r + de−2r + · · · =                    d(e−r )k                c + 6 + 12 + 18 + .......... is an in-
                                                                                10    10     10
                                           k=0                             finite geometric series with a = c and
       which is a geometric series with                                          1
       a = d and e−r = e−r < 1 if r > 0.                                   r= 6 <1,
                                                                                10
            ∞                                                                                  c
                      k      d                                             it converges to
       So      d(e−r) =           .                                                        1 − 10−6
                          1 − e−r
           k=0                                                             where
       If r = .1,                                                                           x     999−x
                                                                                 c         103 + 106
                                                                                    −6 =
        ∞
                           d                                                 1 − 10          1 − 10−6
            d(e−.1 )k =          =2
                        1 − e−.1                                               1000x + 999 − x     999x + 999
       k=0
       so d = 2(1 − .905) ≈ .19                                             =                   =
                                                                                    999999           999999
                                      ∞                k                       x+1
                    15                             1                        =
 54. 30 + 15 +         + ··· =             30                                   1001
                    2                              2
                                     k=0
            30
       =         = 60 miles.                                           (c) Let x + 1 be a positive inte-
         1 − 1/2                                                           ger(assumed) less than 1000 such that
       The bikes meet after 1 hour. In that time, a
                                                                                   x+1
       fly flying 60 mph will have traveled 60 miles.                                       /11 /13 =
                                                                                     7
       ∞                    k
                       3             100,000                                    x+1
 55.         100,000             =           = $400,000.                    =
                       4             1 − 3/4                                    1001
       k=0                                                                           c        c       c
                                                                            = c + 6 + 12 + 18 + .....
 56. P = ce−r + ce−2r + ce−3r + . . .                                               10     10       10
           ∞               ∞
                                           k                                     x     999 − x       x     999 − x
       =         ce−kr =         c e−r                                      = 3+            6    + 9+
                                                                                10       10         10       1012
           k=1             k=1
                                                                                   x     999 − x
           ce−r      c                                                         + 15 +              +···
       =         = r                                                             10         1018
         1 − e−r  e −1                                                     First three digits are given by the term
                                                                             x
 57. Since 0 < p < 1, therefore −1 < −p < 0, and                                 only. Therefore the first three dig-
     0 < 1 − p < 1. Thus, the given series is geo-                         103
                                                                                   x+1
     metric with common ratio r = 1 − p, and so                            its of         are the digits of x. Thus
                                                                                    103
     converges:                                                            here the digits of the number on the left
       ∞                             ∞
                                                                           side repeat after every 6 digits and first
             p(1 − p)n−1 = p             rn−1
                                                                           three digits ( i.e x) is less by one than the
       n=1                        n=1
           1         p       p                                             assumed number (x + 1) and last three
       =p     =             = = 1.                                         digits are 9’s complements of x.
          1−r   1 − (1 − p)  p
Ism et chapter_8
Ism et chapter_8
Ism et chapter_8
Ism et chapter_8
Ism et chapter_8
Ism et chapter_8
Ism et chapter_8
Ism et chapter_8
Ism et chapter_8
Ism et chapter_8
Ism et chapter_8
Ism et chapter_8
Ism et chapter_8
Ism et chapter_8
Ism et chapter_8
Ism et chapter_8
Ism et chapter_8
Ism et chapter_8
Ism et chapter_8
Ism et chapter_8
Ism et chapter_8
Ism et chapter_8
Ism et chapter_8
Ism et chapter_8
Ism et chapter_8
Ism et chapter_8
Ism et chapter_8
Ism et chapter_8
Ism et chapter_8
Ism et chapter_8
Ism et chapter_8
Ism et chapter_8
Ism et chapter_8
Ism et chapter_8
Ism et chapter_8
Ism et chapter_8
Ism et chapter_8
Ism et chapter_8
Ism et chapter_8
Ism et chapter_8
Ism et chapter_8
Ism et chapter_8
Ism et chapter_8
Ism et chapter_8
Ism et chapter_8
Ism et chapter_8
Ism et chapter_8
Ism et chapter_8
Ism et chapter_8
Ism et chapter_8
Ism et chapter_8
Ism et chapter_8
Ism et chapter_8
Ism et chapter_8
Ism et chapter_8
Ism et chapter_8
Ism et chapter_8
Ism et chapter_8

More Related Content

Similar to Ism et chapter_8

Wikimedia Conference 2009 presentation
Wikimedia Conference 2009 presentationWikimedia Conference 2009 presentation
Wikimedia Conference 2009 presentationYu Suzuki
 
Brief survey on Three-Dimensional Displays
Brief survey on Three-Dimensional DisplaysBrief survey on Three-Dimensional Displays
Brief survey on Three-Dimensional DisplaysTaufiq Widjanarko
 
Signal Processing Course : Wavelets
Signal Processing Course : WaveletsSignal Processing Course : Wavelets
Signal Processing Course : WaveletsGabriel Peyré
 
An introduction to the species-people correlation
An introduction to the species-people correlationAn introduction to the species-people correlation
An introduction to the species-people correlationMarco Pautasso
 
Talk data sciencemeetup
Talk data sciencemeetupTalk data sciencemeetup
Talk data sciencemeetupdatasciencenl
 
Why we don’t know how many colors there are
Why we don’t know how many colors there areWhy we don’t know how many colors there are
Why we don’t know how many colors there areJan Morovic
 
CAE REPORT
CAE REPORTCAE REPORT
CAE REPORTdayahisa
 
Class 30: Sex, Religion, and Politics
Class 30: Sex, Religion, and PoliticsClass 30: Sex, Religion, and Politics
Class 30: Sex, Religion, and PoliticsDavid Evans
 
Shape contexts
Shape contextsShape contexts
Shape contextshuebesao
 
Hmm tutorial
Hmm tutorialHmm tutorial
Hmm tutorialPiyorot
 
Design &amp; Simulation of Cargo Stabilization System
Design &amp; Simulation of Cargo Stabilization SystemDesign &amp; Simulation of Cargo Stabilization System
Design &amp; Simulation of Cargo Stabilization SystemHasnain Ali
 

Similar to Ism et chapter_8 (20)

Wikimedia Conference 2009 presentation
Wikimedia Conference 2009 presentationWikimedia Conference 2009 presentation
Wikimedia Conference 2009 presentation
 
Math IA
Math IAMath IA
Math IA
 
Brief survey on Three-Dimensional Displays
Brief survey on Three-Dimensional DisplaysBrief survey on Three-Dimensional Displays
Brief survey on Three-Dimensional Displays
 
Signal Processing Course : Wavelets
Signal Processing Course : WaveletsSignal Processing Course : Wavelets
Signal Processing Course : Wavelets
 
An introduction to the species-people correlation
An introduction to the species-people correlationAn introduction to the species-people correlation
An introduction to the species-people correlation
 
Talk data sciencemeetup
Talk data sciencemeetupTalk data sciencemeetup
Talk data sciencemeetup
 
Ism et chapter_12
Ism et chapter_12Ism et chapter_12
Ism et chapter_12
 
Ism et chapter_12
Ism et chapter_12Ism et chapter_12
Ism et chapter_12
 
DCT_TR802
DCT_TR802DCT_TR802
DCT_TR802
 
DCT_TR802
DCT_TR802DCT_TR802
DCT_TR802
 
DCT_TR802
DCT_TR802DCT_TR802
DCT_TR802
 
Why we don’t know how many colors there are
Why we don’t know how many colors there areWhy we don’t know how many colors there are
Why we don’t know how many colors there are
 
Ml01
Ml01Ml01
Ml01
 
CAE REPORT
CAE REPORTCAE REPORT
CAE REPORT
 
Mark 20121024
Mark 20121024Mark 20121024
Mark 20121024
 
Class 30: Sex, Religion, and Politics
Class 30: Sex, Religion, and PoliticsClass 30: Sex, Religion, and Politics
Class 30: Sex, Religion, and Politics
 
Shape contexts
Shape contextsShape contexts
Shape contexts
 
Hmm tutorial
Hmm tutorialHmm tutorial
Hmm tutorial
 
Design &amp; Simulation of Cargo Stabilization System
Design &amp; Simulation of Cargo Stabilization SystemDesign &amp; Simulation of Cargo Stabilization System
Design &amp; Simulation of Cargo Stabilization System
 
December 7, Projects
December 7, ProjectsDecember 7, Projects
December 7, Projects
 

More from Drradz Maths

More from Drradz Maths (20)

Figures
FiguresFigures
Figures
 
Formulas
FormulasFormulas
Formulas
 
Revision 7.1 7.3
Revision 7.1 7.3Revision 7.1 7.3
Revision 7.1 7.3
 
Tutorial 9
Tutorial 9Tutorial 9
Tutorial 9
 
Tutorial 8
Tutorial 8Tutorial 8
Tutorial 8
 
MTH3101 Tutor 7 lagrange multiplier
MTH3101  Tutor 7 lagrange multiplierMTH3101  Tutor 7 lagrange multiplier
MTH3101 Tutor 7 lagrange multiplier
 
Tutorial 6 en.mughti important
Tutorial 6 en.mughti importantTutorial 6 en.mughti important
Tutorial 6 en.mughti important
 
Figures
FiguresFigures
Figures
 
Formulas
FormulasFormulas
Formulas
 
Figures
FiguresFigures
Figures
 
First_attachment MTH3101
First_attachment MTH3101First_attachment MTH3101
First_attachment MTH3101
 
Tutorial 9 mth 3201
Tutorial 9 mth 3201Tutorial 9 mth 3201
Tutorial 9 mth 3201
 
Tutorial 8 mth 3201
Tutorial 8 mth 3201Tutorial 8 mth 3201
Tutorial 8 mth 3201
 
Tutorial 7 mth 3201
Tutorial 7 mth 3201Tutorial 7 mth 3201
Tutorial 7 mth 3201
 
Tutorial 6 mth 3201
Tutorial 6 mth 3201Tutorial 6 mth 3201
Tutorial 6 mth 3201
 
Tutorial 5 mth 3201
Tutorial 5 mth 3201Tutorial 5 mth 3201
Tutorial 5 mth 3201
 
Tutorial 4 mth 3201
Tutorial 4 mth 3201Tutorial 4 mth 3201
Tutorial 4 mth 3201
 
Tutorial 2, 2(e), no. 7
Tutorial 2,  2(e),  no. 7Tutorial 2,  2(e),  no. 7
Tutorial 2, 2(e), no. 7
 
Tutorial 3 mth 3201
Tutorial 3 mth 3201Tutorial 3 mth 3201
Tutorial 3 mth 3201
 
Tutorial 2 mth 3201
Tutorial 2 mth 3201Tutorial 2 mth 3201
Tutorial 2 mth 3201
 

Recently uploaded

Interpreting the Secrets of Milan Night Chart
Interpreting the Secrets of Milan Night ChartInterpreting the Secrets of Milan Night Chart
Interpreting the Secrets of Milan Night ChartChart Kalyan
 
Spain Vs Italy 20 players confirmed for Spain's Euro 2024 squad, and three po...
Spain Vs Italy 20 players confirmed for Spain's Euro 2024 squad, and three po...Spain Vs Italy 20 players confirmed for Spain's Euro 2024 squad, and three po...
Spain Vs Italy 20 players confirmed for Spain's Euro 2024 squad, and three po...World Wide Tickets And Hospitality
 
大学假文凭《原版英国Imperial文凭》帝国理工学院毕业证制作成绩单修改
大学假文凭《原版英国Imperial文凭》帝国理工学院毕业证制作成绩单修改大学假文凭《原版英国Imperial文凭》帝国理工学院毕业证制作成绩单修改
大学假文凭《原版英国Imperial文凭》帝国理工学院毕业证制作成绩单修改atducpo
 
CALL ON ➥8923113531 🔝Call Girls Saharaganj Lucknow best Female service 🦺
CALL ON ➥8923113531 🔝Call Girls Saharaganj Lucknow best Female service  🦺CALL ON ➥8923113531 🔝Call Girls Saharaganj Lucknow best Female service  🦺
CALL ON ➥8923113531 🔝Call Girls Saharaganj Lucknow best Female service 🦺anilsa9823
 
ALL NFL NETWORK CONTACTS- April 29, 2024
ALL NFL NETWORK CONTACTS- April 29, 2024ALL NFL NETWORK CONTACTS- April 29, 2024
ALL NFL NETWORK CONTACTS- April 29, 2024Brian Slack
 
Serbia vs England Vanja Milinkovic-Savic's Stellar Performance in Euro 2024 P...
Serbia vs England Vanja Milinkovic-Savic's Stellar Performance in Euro 2024 P...Serbia vs England Vanja Milinkovic-Savic's Stellar Performance in Euro 2024 P...
Serbia vs England Vanja Milinkovic-Savic's Stellar Performance in Euro 2024 P...Eticketing.co
 
Plan d'orientations stratégiques rugby féminin
Plan d'orientations stratégiques rugby fémininPlan d'orientations stratégiques rugby féminin
Plan d'orientations stratégiques rugby fémininThibaut TATRY
 
Chennai Call Girls Anna Nagar Phone 🍆 8250192130 👅 celebrity escorts service
Chennai Call Girls Anna Nagar Phone 🍆 8250192130 👅 celebrity escorts serviceChennai Call Girls Anna Nagar Phone 🍆 8250192130 👅 celebrity escorts service
Chennai Call Girls Anna Nagar Phone 🍆 8250192130 👅 celebrity escorts servicevipmodelshub1
 
ppt on Myself, Occupation and my Interest
ppt on Myself, Occupation and my Interestppt on Myself, Occupation and my Interest
ppt on Myself, Occupation and my InterestNagaissenValaydum
 
Atlanta Dream Exec Dan Gadd on Driving Fan Engagement and Growth, Serving the...
Atlanta Dream Exec Dan Gadd on Driving Fan Engagement and Growth, Serving the...Atlanta Dream Exec Dan Gadd on Driving Fan Engagement and Growth, Serving the...
Atlanta Dream Exec Dan Gadd on Driving Fan Engagement and Growth, Serving the...Neil Horowitz
 
Indian Premiere League 2024 by livecricline
Indian Premiere League 2024 by livecriclineIndian Premiere League 2024 by livecricline
Indian Premiere League 2024 by livecriclineLive Cric Line
 
TAM Sports_IPL 17 Till Match 37_Celebrity Endorsement _Report.pdf
TAM Sports_IPL 17 Till Match 37_Celebrity Endorsement _Report.pdfTAM Sports_IPL 17 Till Match 37_Celebrity Endorsement _Report.pdf
TAM Sports_IPL 17 Till Match 37_Celebrity Endorsement _Report.pdfSocial Samosa
 
08448380779 Call Girls In IIT Women Seeking Men
08448380779 Call Girls In IIT Women Seeking Men08448380779 Call Girls In IIT Women Seeking Men
08448380779 Call Girls In IIT Women Seeking MenDelhi Call girls
 
JORNADA 4 LIGA MURO 2024TUXTEPEC1234.pdf
JORNADA 4 LIGA MURO 2024TUXTEPEC1234.pdfJORNADA 4 LIGA MURO 2024TUXTEPEC1234.pdf
JORNADA 4 LIGA MURO 2024TUXTEPEC1234.pdfArturo Pacheco Alvarez
 
08448380779 Call Girls In Karol Bagh Women Seeking Men
08448380779 Call Girls In Karol Bagh Women Seeking Men08448380779 Call Girls In Karol Bagh Women Seeking Men
08448380779 Call Girls In Karol Bagh Women Seeking MenDelhi Call girls
 
Albania Vs Spain Albania is Loaded with Defensive Talent on their Roster.docx
Albania Vs Spain Albania is Loaded with Defensive Talent on their Roster.docxAlbania Vs Spain Albania is Loaded with Defensive Talent on their Roster.docx
Albania Vs Spain Albania is Loaded with Defensive Talent on their Roster.docxWorld Wide Tickets And Hospitality
 
Tableaux 9ème étape circuit fédéral 2024
Tableaux 9ème étape circuit fédéral 2024Tableaux 9ème étape circuit fédéral 2024
Tableaux 9ème étape circuit fédéral 2024HechemLaameri
 

Recently uploaded (20)

Interpreting the Secrets of Milan Night Chart
Interpreting the Secrets of Milan Night ChartInterpreting the Secrets of Milan Night Chart
Interpreting the Secrets of Milan Night Chart
 
Spain Vs Italy 20 players confirmed for Spain's Euro 2024 squad, and three po...
Spain Vs Italy 20 players confirmed for Spain's Euro 2024 squad, and three po...Spain Vs Italy 20 players confirmed for Spain's Euro 2024 squad, and three po...
Spain Vs Italy 20 players confirmed for Spain's Euro 2024 squad, and three po...
 
大学假文凭《原版英国Imperial文凭》帝国理工学院毕业证制作成绩单修改
大学假文凭《原版英国Imperial文凭》帝国理工学院毕业证制作成绩单修改大学假文凭《原版英国Imperial文凭》帝国理工学院毕业证制作成绩单修改
大学假文凭《原版英国Imperial文凭》帝国理工学院毕业证制作成绩单修改
 
CALL ON ➥8923113531 🔝Call Girls Saharaganj Lucknow best Female service 🦺
CALL ON ➥8923113531 🔝Call Girls Saharaganj Lucknow best Female service  🦺CALL ON ➥8923113531 🔝Call Girls Saharaganj Lucknow best Female service  🦺
CALL ON ➥8923113531 🔝Call Girls Saharaganj Lucknow best Female service 🦺
 
ALL NFL NETWORK CONTACTS- April 29, 2024
ALL NFL NETWORK CONTACTS- April 29, 2024ALL NFL NETWORK CONTACTS- April 29, 2024
ALL NFL NETWORK CONTACTS- April 29, 2024
 
Serbia vs England Vanja Milinkovic-Savic's Stellar Performance in Euro 2024 P...
Serbia vs England Vanja Milinkovic-Savic's Stellar Performance in Euro 2024 P...Serbia vs England Vanja Milinkovic-Savic's Stellar Performance in Euro 2024 P...
Serbia vs England Vanja Milinkovic-Savic's Stellar Performance in Euro 2024 P...
 
Plan d'orientations stratégiques rugby féminin
Plan d'orientations stratégiques rugby fémininPlan d'orientations stratégiques rugby féminin
Plan d'orientations stratégiques rugby féminin
 
Chennai Call Girls Anna Nagar Phone 🍆 8250192130 👅 celebrity escorts service
Chennai Call Girls Anna Nagar Phone 🍆 8250192130 👅 celebrity escorts serviceChennai Call Girls Anna Nagar Phone 🍆 8250192130 👅 celebrity escorts service
Chennai Call Girls Anna Nagar Phone 🍆 8250192130 👅 celebrity escorts service
 
ppt on Myself, Occupation and my Interest
ppt on Myself, Occupation and my Interestppt on Myself, Occupation and my Interest
ppt on Myself, Occupation and my Interest
 
Atlanta Dream Exec Dan Gadd on Driving Fan Engagement and Growth, Serving the...
Atlanta Dream Exec Dan Gadd on Driving Fan Engagement and Growth, Serving the...Atlanta Dream Exec Dan Gadd on Driving Fan Engagement and Growth, Serving the...
Atlanta Dream Exec Dan Gadd on Driving Fan Engagement and Growth, Serving the...
 
Call Girls Service Noida Extension @9999965857 Delhi 🫦 No Advance VVIP 🍎 SER...
Call Girls Service Noida Extension @9999965857 Delhi 🫦 No Advance  VVIP 🍎 SER...Call Girls Service Noida Extension @9999965857 Delhi 🫦 No Advance  VVIP 🍎 SER...
Call Girls Service Noida Extension @9999965857 Delhi 🫦 No Advance VVIP 🍎 SER...
 
Call Girls In Vasundhara 📱 9999965857 🤩 Delhi 🫦 HOT AND SEXY VVIP 🍎 SERVICE
Call Girls In Vasundhara 📱  9999965857  🤩 Delhi 🫦 HOT AND SEXY VVIP 🍎 SERVICECall Girls In Vasundhara 📱  9999965857  🤩 Delhi 🫦 HOT AND SEXY VVIP 🍎 SERVICE
Call Girls In Vasundhara 📱 9999965857 🤩 Delhi 🫦 HOT AND SEXY VVIP 🍎 SERVICE
 
Indian Premiere League 2024 by livecricline
Indian Premiere League 2024 by livecriclineIndian Premiere League 2024 by livecricline
Indian Premiere League 2024 by livecricline
 
TAM Sports_IPL 17 Till Match 37_Celebrity Endorsement _Report.pdf
TAM Sports_IPL 17 Till Match 37_Celebrity Endorsement _Report.pdfTAM Sports_IPL 17 Till Match 37_Celebrity Endorsement _Report.pdf
TAM Sports_IPL 17 Till Match 37_Celebrity Endorsement _Report.pdf
 
08448380779 Call Girls In IIT Women Seeking Men
08448380779 Call Girls In IIT Women Seeking Men08448380779 Call Girls In IIT Women Seeking Men
08448380779 Call Girls In IIT Women Seeking Men
 
JORNADA 4 LIGA MURO 2024TUXTEPEC1234.pdf
JORNADA 4 LIGA MURO 2024TUXTEPEC1234.pdfJORNADA 4 LIGA MURO 2024TUXTEPEC1234.pdf
JORNADA 4 LIGA MURO 2024TUXTEPEC1234.pdf
 
08448380779 Call Girls In Karol Bagh Women Seeking Men
08448380779 Call Girls In Karol Bagh Women Seeking Men08448380779 Call Girls In Karol Bagh Women Seeking Men
08448380779 Call Girls In Karol Bagh Women Seeking Men
 
Albania Vs Spain Albania is Loaded with Defensive Talent on their Roster.docx
Albania Vs Spain Albania is Loaded with Defensive Talent on their Roster.docxAlbania Vs Spain Albania is Loaded with Defensive Talent on their Roster.docx
Albania Vs Spain Albania is Loaded with Defensive Talent on their Roster.docx
 
Call Girls 🫤 Malviya Nagar ➡️ 9999965857 ➡️ Delhi 🫦 Russian Escorts FULL ENJOY
Call Girls 🫤 Malviya Nagar ➡️ 9999965857  ➡️ Delhi 🫦  Russian Escorts FULL ENJOYCall Girls 🫤 Malviya Nagar ➡️ 9999965857  ➡️ Delhi 🫦  Russian Escorts FULL ENJOY
Call Girls 🫤 Malviya Nagar ➡️ 9999965857 ➡️ Delhi 🫦 Russian Escorts FULL ENJOY
 
Tableaux 9ème étape circuit fédéral 2024
Tableaux 9ème étape circuit fédéral 2024Tableaux 9ème étape circuit fédéral 2024
Tableaux 9ème étape circuit fédéral 2024
 

Ism et chapter_8

  • 1. (c) 2 1.8 1.6 Chapter 8 1.4 1.2 1 Infinite Series 0.8 0 2 4 6 8 10 12 x n 1 7. (a) lim = lim 1 =1 n→∞ n + 1 n→∞ 1 + 8.1 Sequences of Real n Numbers (b) As n gets large, n/(n + 1) gets close to 1. 3 5 7 9 11 (c) 1. 1, , , , , 4 9 16 25 36 0.9 3 1 3 3 1 3 2. , , , , , 5 2 7 8 3 10 0.8 2 1 1 1 3. 4, 2, , , , 0.7 3 6 30 180 1 2 3 4 5 6 0.6 4. − , , − , , − , 2 3 4 5 6 7 0.5 1 0 2 4 6 8 10 12 5. (a) lim 3 = 0 x n→∞ n (b) As n gets large, n3 gets large, so 1/n3 goes 2n + 1 1 to 0. 8. (a) lim = lim 2 + =2 n→∞ n n→∞ n (c) (b) As n gets large, (2n + 1)/n gets close to 1 2. 0.8 (c) 0.6 0.4 0.2 0 0 2 4 6 8 10 12 x 2 6. (a) lim √ = 0 n→∞ n √ 2 (b) As n gets large, n gets large, so √ goes n 1 to 0. 9. (a) In Q.(5), an = n3 443
  • 2. 444 CHAPTER 8. INFINITE SERIES 1.0 3.0 2.5 0.75 2.0 0.5 1.5 1.0 0.25 0.5 0.0 0.0 2 4 6 8 10 5 10 15 20 As n gets larger and larger, the terms As n gets larger and larger, the terms 1 of the sequence an = n3 approach to 0. of the sequence an = 2n+1 approach 2. n Hence, an converges to 0. Hence, the sequence an = 2n+1 converges n to 2. 2 (b) In Q.(6), an = √ 10. n 1.0 0.9 0.8 0.5 0.7 0.0 0.6 0 10 20 30 40 0.5 −0.5 0.4 0.3 −1.0 10 20 30 40 50 As n gets larger, the terms of the sequence As n gets larger and larger, the terms n nπ nπ 2 an = sin + cos approach to 1 of the sequence an = √n approach to 0. n+1 2 2 2 or -1. When n is of the form 4m or 4m + 1, the Hence, the sequence an = √n converges terms of the sequence an approach 1. When to 0. n is of the form 4m + 2 or 4m + 3, the terms n of the sequence an approach -1. Therefore, an (c) In Q.(7), an = does not convergent. n+1 1 3n2 + 1 3+ n2 3 11. lim 2−1 = lim 1 = n→∞ 2n n→∞ 2 − 2 0.9 n2 1 5n3 − 1 5− 3 0.8 12. lim = lim n =5 n→∞ 2n3 + 1 n→∞ 1 2 0.7 2+ 3 n 1 0.6 n2 + 1 n+ n 13. lim = lim =∞ n→∞ n + 1 n→∞ 1 + 1 n 0.5 5 10 15 20 25 30 1 1 n2 + 1 + 3 As n gets larger and larger, the terms 14. lim 3 = lim n n = 0 n→∞ n + 1 n→∞ 1 n of the sequence an = n+1 approach 1. 1+ 3 n n Hence, the sequence an = n+1 converges 2 to 1. n+2 1+ 15. lim (−1)n = lim (−1)n n 1 n→∞ 3n − 1 n→∞ 3− n 2n + 1 1 (d) In Q.(8), an = = ± , the limit does not exist; diverges n 3
  • 3. 8.1. SEQUENCES OF REAL NUMBERS 445 n 16. Diverges. Even terms converge to 1 and odd = lim √ terms converge to −1. n→∞ n2 + n + n 1 1 = lim = n+2 n n+2 n→∞ 1 + 1/n + 1 2 17. lim (−1) 2 = lim 2 n→∞ n +4 n→∞ n + 4 1 + 22 27. lim [ln (2n + 1) − ln n] = lim n n = 0 n→∞ n→∞ 1 + 4 2n + 1 n2 (−1)2 (n + 2) = lim ln Hence lim =0 n→∞ n n→∞ n2 + 4 1 = lim ln 2 + = ln 2 18. Diverges. Even terms are 1 and odd terms are n→∞ n −1. nπ 28. When n is even number, cos = 1 and x 1 2 19. lim x = lim x by l’Hopital’s Rule nπ 2n − 1 2n − 1 x→∞ e x→∞ e lim cos = lim = 2. 1 n n→∞ 2 n+2 n→∞ n + 2 and lim x = 0, so by Theorem 1.2 lim n = nπ x→∞ e n→∞ e When n is odd number, cos = 0 and 2 0 nπ 2n − 1 lim cos = 0. 1 cos n 1 n→∞ 2 n+2 20. Note that − n ≤ n ≤ n and Therefore, the limit does not exist. e e e 1 cos n lim = 0, so lim =0 29. By L’Hopital Rule, n→∞ en n→∞ en by the Squeeze Theorem. n3 + 1 3n2 6n lim n = lim n = lim n 1 2 n→∞ e n→∞ e n→∞ e en + 2 n + e2n 6 21. lim = lim e 1 =0 = lim n = 0. n→∞ e2n − 1 n→∞ 1 − 2n e n→∞ e 22. For n ≥ 1, en + 1 < 2en , so 30. By L’Hopital Rule, n 3n 1 3n 1 3 ln n 1 > · n = · . lim √ = lim n en + 1 2 e 2 e n→∞ n + 1 n→∞ n √1 3 1 3 2 n+1 Since > 1, lim = ∞; √ e n→∞ 2 e 2 n+1 3n = lim so lim n =∞ n→∞ n n→∞ e + 1 1 1 = lim 2 + = 0. x2x x x n→∞ n n2 23. lim = lim x = lim x x→∞ 3x x→∞ 3 x→∞ 3 −1 cos n 1 2x 2 31. −1 ≤ cos n ≤ 1 ⇒ ≤ ≤ 2 1 n2 n2 n = lim = 0, by l’Hopital’s Rule, −1 1 x→∞ 3 x 3 for all n, and lim 2 = lim 2 = 0 n→∞ n n→∞ n ln 2 2 so by the Squeeze Theorem, 3 x cos n since lim = ∞. lim =0 n→∞ n2 x→∞ 2 n n2 1 cos πn 1 1 Hence lim n = 0, by Theorem 1.2. 32. Note that − 2 ≤ ≤ 2 and lim 2 = n→∞ 3 n n2 n n→∞ n cos πn n! 1 2 n 1 n n 0, so lim = 0 by the Squeeze Theorem. 24. Since n = · ··· ≥ · = and n→∞ n2 2 2 2 2 2 2 4 n n! 1 1 1 lim = ∞, we have lim n = ∞. 33. 0 ≤ |an | = n ≤ and lim =0 n→∞ 4 n→∞ 2 ne n n→∞ n 1 so by the Squeeze Theorem & Corollary 1.1, 1 sin n 25. lim n sin = lim =1 (−1)n n→∞ n n→∞ n 1 lim =0 n→∞ nen 26. lim n2 + n − n 1 n→∞ √ √ ln x n2 + n − n n2 + n + n 34. lim 2 = lim x = lim 1 = 0, = lim √ x→∞ x x→∞ 2x x→∞ 2x2 n→∞ n2 + n + n by l’Hopital’s Rule.
  • 4. 446 CHAPTER 8. INFINITE SERIES ln n ln n ln n is not a lower bound for S, there exist some Since (−1)n ≤ 2 , we have lim = ∞ n2 n n→∞ n2 aN , of for which aN < G + ε. Since {an }n=1 0, by the Squeeze Theorem and Corollary 1.1. is decreasing sequence, G < an < G + ε for an+1 n+4 n+2 all n ≥ N . That is, G − ε ≤ an ≤ G + ε ∞ 35. = · for all n ≥ N. Hence {an }n=1 converges to G. an n+3 n+3 Therefore bounded and decreasing sequence is n2 + 6n + 8 = 2 < 1 for all n, so convergent. n + 6n + 9 √ an+1 < an for all n, so the sequence is decreas- 48. First we note that a2 = 3 + 2 3 > a1 . As- ing. sume that ak > ak−1 , then n n−1 √ √ 36. Since an+1 − an = − ak+1 = 3 + 2 ak > 3 + 2 ak−1 = ak n+2 n+1 Then we note that a1 < 3 and if we assume 2 = > 0, the sequence is increas- that ak−1 < 3 then (n + 1)(n + 2) √ √ ing. ak = 3 + 2 ak−1 < 3 + 2 · 3 = 3 We have shown that the sequence is increas- an+1 en+1 n e·n 37. = · n = > 1 for all n, ing with an upper bound, so it converges. The an n+1 e n+1 limit of this sequence should be the solution to ∞ so an+1 > an for all n, so {an }n=1 is increasing. the equation √ 3n+1 x = 3 + 2x, x2 − 2x − 3 = 0 (x + 1)(x − 3) = 0, x = −1, x = 3 Since an+1 (n + 3)! 3 38. Since = = ≤ 1, the se- the limit of the sequence can’t be negative, so an 3n n+3 lim an = 3. (n + 2)! n→∞ quence is decreasing. 49. (a) a1000 ≈ 7.374312390, 3n2 − 2 3n2 − 2 e2 ≈ 7.389056099 39. |an | = 2+1 = 2 b1000 ≈ .135064522, n n +1 3n2 3n2 e−2 ≈ .135335283. < 2 < 2 =3 m n +1 n (b) Let n = , m = nr. r m/r 6n − 1 6n + 18 r n r2 40. |an | = < =6 lim 1 + = lim 1 + n+3 n+3 n→∞ n m→∞ m r m/r 2 sin(n2 ) 1 1 1 41. |an | = ≤ ≤ for n > 1. = lim 1+ = er n+1 n+1 2 m→∞ m/r2 as m/r2 → ∞ 42. |an | = e1/n ≤ e1 = e n−4 43. an = −(−2) 50. (a) For the first 8 terms we get: 1 a1 = 1, 44. an = 2n−1 a2 = 2.5, 2n−1 a3 = 2.05, 45. an = n2 a4 = 2.000609756, (−1)n+1 n a5 = 2.000000093, 46. an = (n+1)2 a6 = 2.000000000, ∞ a7 = 2.000000000, 47. Suppose that {an }n=1 is decreasing and a8 = 2.000000000 bounded sequence. Then for some M > 0, 1 4 |an | < M for all n. Let S be the set con- The equation L = L+ has two taining all of the terms of the sequence, S = 2 L solutions, L = 2 and L = −2. Since the {a1 , a2 , a3 , ...} . By Completeness Axiom, S terms of the sequence are positive, we dis- must have a greatest lower bound say G. That card the negative solution. Thus the limit is, G is the greatest number for which an > G must be L = 2. , for all n . Therefore, for any number ε > 0,G + ε > G and so,G + ε is not a lower bound, (b) Assuming the limit exists, and letting L = since G is greatest lower bound. Since G + ε lim an+1 = lim an , we have n→∞ n→∞
  • 5. 8.1. SEQUENCES OF REAL NUMBERS 447 1 c converges for p < −1 and p ≥ 1. L= L+ ⇒ 2L2 = L2 + c 2 L √ 1 ⇒ L2 = c ⇒ L = c. (b) p > 0 ⇒ lim =0 n→∞ np 1 51. Use induction to show that an+1 > an . First √ p = 0 ⇒ lim p = 1 n→∞ n note that a2 = 2, and since a2 = 2 + 2, 1 2 1 it follows that a2 > a2 , so a2 > a1 . Thus, p < 0 ⇒ lim p does not exist 2 1 n→∞ n the statement is true for n = 1. Now assume Therefore, the sequence an = 1/np that the statement is true for n = k (that is, converges for p ≥ 0. ak+1 > ak ), and show that the statement is true for n = k + 1. First note that 1 √ 53. an = (1 + 2 + 3 + · · · + n) ak+2 = 2 + ak+1 n2 √ 1 n(n + 1) n+1 1 1 a2 = 2 + ak+1 = 2 = = 1+ k+2 2 √ n 2 2n 2 n ak+2 − 2 = ak+1 1 1 1 2 lim an = lim 1 + = a2 − 2 = ak+1 k+2 n→∞ 2 n→∞ n 2 The previous equation is also valid if we replace Thus, the sequence an converges to 1/2. 2 k by k − 1: a2 − 2 = ak k+1 Note that n 1 Since ak+1 > ak , it follows that 1k 2 xdx = lim ak+1 > a2 − 2 and therefore k+1 0 n→∞ k=1 nn 2 2 Therefore the sequence an converges to a2 − 2 > a2 − 2 k+2 k+1 1 a2 − 2 > a2 − 2 k+2 k+1 xdx. a2 > a2 , ak+2 > ak+1 k+2 k+1 0 Thus, by induction, an is increasing. Now we’ll n prove that an < 2 by induction. First note 1 54. lim an = lim that a1 < 2, and assume that ak < 2. Then n→∞ n→∞ n+k k=1 √ n ak+1 = 2 + ak 1 n √ 2 = lim a2 − 2 = ak , a2 − 2 = ak k+1 k+1 n→∞ nn+k 2 √ k=1 a2 − 2 < 2, a2 − 2 < 2 k+1 n 1 1 √ 2 k+1 a2 < 2 + 2, ak+1 < 4 = lim k+1 n→∞ n 1 + k/n ak+1 < 2 Thus, by induction, an < 2. k=1 1 Since an is increasing and bounded above by 1 1 = dx = ln |1 + x||0 2, an converges. To estimate the limit, we’ll √ 0 1+x approximate the solution of x = √ 2 + x: = ln 2 − ln 1 = ln 2 x2 = 2 + x, (x2 − 2)2 = x Therefore the sequence converges. x4 − 4x2 + 4 = x, 0 = x4 − 4x2 − x + 4 0 = (x − 1)(x3 + x2 − 3x − 4) √ 55. (a) Begin by joining the centers of circles C1 Since an > 2, it follows that x = 1. and C2 with a line segment. The length Therefore, 0 = x3 + x2 − 3x − 4. Using a CAS, of this line segment is the sum of the radii the solution is x ≈ 1.8312. of the two circles, which is r1 + r2 . Thus, 1 the squared of the length of the line seg- 52. (a) p > 1 ⇒ lim =0 ment is (r1 +r2 )2 . Now, the coordinates of n→∞ pn 1 the centers of the circles are (c1 , r1 ) and p = 1 ⇒ lim n = 1 (c2 , r2 ). Using the formula for the dis- n→∞ p 1 tance between two points, the square of 0 < p < 1 ⇒ lim n does not exist the length of the line segment joining the n→∞ p 1 two centers is (c2 −c1 )2 +(r2 −r1 )2 . Equat- − 1 < p < 0 ⇒ lim n does not exist ing the two expressions, we get n→∞ p 1 (c2 − c1 )2 + (r2 − r1 )2 = (r1 + r2 )2 p = −1 ⇒ lim n does not exist Expanding and simplifying this relation- n→∞ p 1 ship, we get p < −1 ⇒ lim n = 0 n→∞ p (c2 − c1 )2 = (r1 + r2 )2 − (r2 − r1 )2 2 2 2 2 Therefore, the sequence an = 1/pn = r1 + 2r1 r2 + r2 − (r2 − 2r1 r2 + r1 )
  • 6. 448 CHAPTER 8. INFINITE SERIES = 4r1 r2 57. Given that s2n = 2 − 4 − sn 2 and √ |c2 − c1 | = 2 r1 r2 s6 = 1. Therefore, (b) The same reasoning applied to the other two pairs of centers yields analogous re- S12 = 2− 4 − S6 2 sults. Without going through the motions √ again, you can simply take the results = 2− 4 − 12 = 2− 3 above and first replace all subscripts “2” by “3” to get the results for circles C1 S24 = 2− 4 − S12 2 and C3 . Then take these new results and √ replace all subscripts “1” by “2” to get = 2− 4− 2− 3 the results for circles C2 and C3 . The results are √ (c3 − c1 )2 + (r3 − r1 )2 = (r1 + r3 )2 = 2− 2+ 3 √ |c3 − c1 | = 2 r1 r3 (c3 − c2 )2 + (r3 − r2 )2 = (r2 + r3 )2 S48 = 2− 4 − S24 2 √ |c3 − c2 | = 2 r2 r3 √ = 2− 4− 2− 2+ 3 (c) Finally, |c1 − c2 | = |c1 − c3 | + |c3 − c2 | √ √ √ 2 r1 r2 = 2 r1 r3 + 2 r2 r3 √ √ √ √ √ = 2− 2+ 2+ 3 ≈ 0.130806 r1 r2 = √ 3 ( r1 + r2 ) r √ r1 r2 As S6 = 1, the length of each side of the regu- r3 = √ √ r1 + r2 lar hexagon inscribed in the circle is 1 and the (d) Given that r1 = r2 = 1, from Part(c), we corresponding angle subtended at the centre π have √ of the circle is , which implies the radius of √ r1 r2 1 3 r3 = √ √ = the circle is 1. As n increases the length of the r1 + r2 √ 2 arc is approximately equal to the length of the √ r2 r3 1/2 1 2π r4 = √ √ = = side of the regular n-gon. Therefore, Sn ≈ r2 + r3 √ 1 + 1/2 3 n π √ r3 r4 1/6 1 . Thus, S48 ≈ . r5 = √ √ = = 24 r3 + r4 1/2 + 1/3 5 1 1 1 The pattern is √ = √ +√ rn rn−1 rn−2 Hence if Fn is the n-th Fibonacci number, 2 then rn = 1/Fn . 56. (a) The distance between the two points (0, c) and (x0 , y0 ), where y0 = x2 , is r, 0 x2 + (x2 − c)2 = r 0 0 x2 + x4 − 2cx2 + c2 = r2 0 0 0 y0 + (1 − 2c)y0 + (c2 − r2 ) = 0 2 n! 1·2·3·4···n 58. 0 < n = We want the solution y0 to the above n n·n·n·n···n equation to be unique, so that 1 2·3·4···n 1 1 = < (1) = (1 − 2c)2 − 4(c2 − r2 ) = 0 n n·n·n···n n n 1 − 4c + 4c2 − 4c2 + 4r2 = 0 n! 1 1 Hence, 0 < n < . 1 − 4c + 4r2 = 0, c = + r2 n n 4 Therefore, n! 1 (b) Following Part(a), the pattern is lim 0 < lim n < lim n→∞ n→∞ n n→∞ n 2 1 1 2 rn + = rn + rn−1 + + rn−1 n! 4 4 0 < lim n < 0 2 2 n→∞ n rn − rn−1 = rn + rn−1 rn − rn−1 = 1 n! lim =0 r1 = 1, r2 = 2, r3 = 3, . . . rn = n n→∞ nn
  • 7. 8.2. INFINITE SERIES 449 2 0.5 61. In the 3rd month, only the adult rabbits have 3 0.5436 newborns, so a3 = 2+1 = 3. In the 4th month, 4 0.5731 only the 2 pairs of adult rabbits from a2 can 5 0.5949 have newborns, so a4 = 3 + 2 = 5. In general, 6 0.6119 an = an−1 + an−2 7 0.6258 8 0.6374 62. Let an and an+1 denote the sizes of the shorter 9 0.6473 and longer sides of the nth rectangle. For ex- 10 0.6559 ample, a1 = 1 and a2 = 2 are the sides of the first rectangle, a2 = 2 and a3 = 3 are the sides 20 0.7065 of the second rectangle, and so on. When a new 30 0.7316 rectangle an ×an+1 is formed, the longer side of 40 0.7476 the previous one, an , becomes the shorter side 50 0.7590 of the new one, and the longer one of the new 60 0.7678 one, an+1 , is the sum of the shorter one and the 70 0.7748 longer one of the previous one. If we express 80 0.7806 that as a formula we get an+1 = an−1 + an , 90 0.7855 which is the property defining the Fibonacci 100 0.7898 sequence. 200 0.8146 300 0.8269 500 0.8403 8.2 Infinite Series 1000 0.8589 ∞ k ln (n!) 1 From the table, lim = 1. 1. 3 is a geometric series with n→∞ ln (nn ) 5 k=0 When x → ∞, slope of the graph y = ln x ap- 1 proaches to 0. Due to this property, the given a = 3 and |r| = < 1, so it converges to 5 limit approaches to 1. 3 15 = 1 − 1/5 4 12 ∞ 59. If side s = 12 and the diameter D = then 1 k n 2. 5 is a geometric series with 3 the number of disks that fit along one side is k=0 12 |r| = 5 > 1, so it diverges. 12 = n. Thus, the total number of disks is n ∞ k 12 12 1 1 · =n·n=n 2 3. − is a geometric series with 12 12 2 3 n n k=0 an = wasted area inbox with n2 disks 1 1 2 a = and |r| = < 1, so it converges to 6 2 3 = 12 · 12 − n2 π = 144 − 36π ≈ 30.9 1/2 3 n = . 1 − (−1/3) 8 ∞ k 60. The answer is a5 = 30 or a5 = 31, depending 1 on the position of the points. 4. 4 is a geometric series with 2 k=0 1 a = 4 and |r| = < 1, so it converges to 2 4 = 8. 1 − 1/2 ∞ 1 k 5. (3) is a geometric series with 2 k=0 |r| = 3 > 1, so it diverges. ∞ k 1 6. 3 − is a geometric series with 2 k=0
  • 8. 450 CHAPTER 8. INFINITE SERIES 1 13. Using partial fractions a = 3 and |r| = < 1 so it converges to n 2 2k + 1 3 Sn = = 2. k 2 (k + 1)2 1 − (−1/2) k=1 n 7. Using partial fractions, 1 1 = − n 4 n 2 2 k2 (k + 1)2 k=1 Sn = = − k(k + 2) k k+2 1 1 1 1 1 k=1 k=1 = 1− + − + − 2 2 2 2 4 4 9 9 16 = 2− + 1− + − 3 4 3 5 1 1 2 2 + ··· + 2 − 2 + ··· + − (n − 1) n n n+2 1 1 2 2 + − =2+1− − n2 (n + 1)2 n+1 n+2 4n + 6 1 =3− 2 and =1− n + 3n + 2 (n + 1)2 4n + 6 1 lim Sn = lim 3 − 2 = 3. and, lim Sn = lim 1− =1 n→∞ n→∞ n + 3n + 2 n→∞ n→∞(n + 1)2 Thus, the series converges to 3. Thus the series converges to 1. 4k ∞ 8. lim = 4 = 0, so the series diverges by 4 k→∞ k + 2 14. th the k -Term Test for Divergence. k(k + 1)(k + 3)(k + 4) k=1 ∞ 3k 4 1 1 9. lim = 3 = 0, so by the k th -Term Test = − k→∞ k + 4 3 k(k + 4) (k + 1)(k + 3) k=1 for Divergence, the series diverges. 4 ∞ 1 4 ∞ 1 = − 3 k(k + 4) 3 (k + 1)(k + 3) 10. This is a telescoping sum: k=1 ∞ k=1 n n 41 1 1 9 3 3 = − Sn = = − 34 k k+4 k(k + 3) k k+3 k=1 k=1 k=1 ∞ 3 3 3 3 3 3 41 1 1 = − + − + ··· + − − − 1 4 2 5 n n+3 32 k+1 k+3 3 3 3 3 3 k=1 =3+ + − − − . ∞ 2 3 n+1 n+2 n+3 1 1 1 ∞ = − 9 3 k k+4 k=1 Thus ∞ k(k + 3) 1 1 1 k=1 − − 3 3 3 3 6 k+1 k+3 = lim 3 + + − − k=1 n→∞ 2 3 n+1 n+2 1 1 1 1 1 1 1 3 = 1+ + + − + − 3 2 3 4 6 2 3 n+3 5 3 3 11 = . = 3+ + = . 9 2 3 2 ∞ ∞ ∞ ∞ 2 1 1 2 11. =2 and from Example 2.7, 15. is a geometric series with k k k ek k=1 k=1 k=1 k=2 ∞ 2 1 1 a = 2 and |r| = < 1,so it converges to diverges, so 2 diverges. e e k 2 2 k=1 = 2 . e 2 1− 1 e −e e ∞ ∞ 4 1 12. = 4 and the harmonic series k+1 k 16. lim |ak | = lim 31/k = 30 = 1 = 0 , So by k=0 k=1 ∞ ∞ k→∞ k→∞ 1 4 the k th Term Test for Divergence, the series diverges, so diverges. k k+1 diverges. k=1 k=0
  • 9. 8.2. INFINITE SERIES 451 ∞ ∞ ∞ 1 1 1 1 gent if |r| = |2c + 1| < 1 . 17. − = − . The 2k k+1 2k k+1 That is, if −1 < 2c + 1 < 1 or −2 < 2c < 0 k=0 k=0 k=0 first series is a convergent geometric series but Therefore, the series converges for the second series is the divergent harmonic se- −1 < c < 0. ries, so the original series diverges. ∞ ∞ ∞ 1 1 1 1 ∞ 18. − k = − 2 2k 3 2k 3k 26. k is a geometric series with a = 2 k=0 k=0 k=0 (c − 3) 1 1 1 k=0 = − = 1 1 − 1/2 1 − 1/3 2 and |r| = . The series converges, if (c − 3) ∞ ∞ ∞ 1 2 1 2 1 |r| = < 1, that is |c − 3| > 1, there- 19. + k = + . c−3 3k 2 3k 2k k=0 k=0 k=0 fore c > 4, c < 2. Thus the series converges for The first series is a geometric series with 1 c < 2 and c > 4. a = 2 and |r| = < 1 so it converges to 3 2 = 3. ∞ 1 − 1/3 c The second series is a geometric series with 27. is divergent for all values of c. Refer 1 k+1 k=0 a = 1 and |r| = < 1, so it converges to Excercies 12 2 1 = 2. 1 − 1/2 ∞ 2 1 28. Two cases possible. Thus + k = 3 + 2 = 5. 3k 2 (i) c = 0: ⇒ given series is k=0 ∞ ∞ ∞ ∞ ∞ 2 1 1 1 1 = 2 which is divergent 20. − k = − (ck + 1) k 4 k 4k k=0 k=0 k=0 k=0 k=0 The second series is a convergent geometric se- ries, but the first series is a divergent harmonic (ii) c = 0: ⇒ given series is ∞ ∞ series, so the series diverges. 2 2 = (ck + 1) c k+ 1 c 3k k=0 k=0 21. lim |ak | = lim =3=0 ∞ k→∞ k→∞ k + 1 2 1 th So by the k -Term Test for Divergence, the = 1 c k+ c k=0 series diverges. ∞ 1 k 3 Which is divergent as 1 22. The limit lim (−1)k 2 does not exists, so k=0 k+ c k→∞ k +1 is divergent. th the series diverges by the k -Term Test for Di- vergence. 23. Since k is positive integer, k is ration num- 5 29. The series appears to converge. ber and can’t be multiple of π. Hence n 1 k n √ lim sin = 0. Therefore , the series di- k=1 k k→∞ 5 verges. 2 1.250000000 4 1.423611111 π 8 1.527422052 24. The limit lim tan−1 (k) = = 0, so by the 16 1.584346533 k→∞ 2 k th Term Test for Divergence, the series di- 32 1.614167263 verges. 64 1.629430501 ∞ 128 1.637152005 k 256 1.641035436 25. 3(2c + 1) is a geometric series with k=0 512 1.642982848 a = 3 and |r| = |2c + 1| . The series is conver- 1024 1.643957981
  • 10. 452 CHAPTER 8. INFINITE SERIES 5 1.6 1.5 4.5 1.4 4 1.3 3.5 1.2 3 0 20 40 60 80 100 1.1 1 32. The series appears to converge. 0 20 40 60 80 100 n 2k n k k=1 30. The series appears to diverge. n 1 1 2 n √ 2 4 k=1 k 3 5.333333333 2 1.707106781 4 6 4 2.784457050 5 6.266666667 8 4.371436800 6 6.355555556 16 6.663994609 7 6.380952381 32 9.941512173 8 6.387301587 64 14.60206410 9 6.388712522 128 21.21122790 10 6.388994709 256 30.57088534 512 43.81657304 1024 62.55526935 6 5 12 10 4 8 3 6 2 4 10 20 30 40 50 2 ∞ 10 20 30 40 50 33. (a) Assume ak converges to L. k=1 Then for any m, ∞ m−1 ∞ 31. The series appears to converge. L= ak = ak + ak k=1 k=1 k=m n ∞ 1 n √ = Sm−1 + ak . k=1 k k=m 2 4.500000000 ∞ 4 5.125000000 So ak = L − Sm−1 , k=m 8 5.154836310 and thus converges. 16 5.154845485 ∞ ∞ 32 5.154845485 (b) Since ak = (a1 + · · · + am−1 ) + ak 1024 5.154845485 k=1 k=m
  • 11. 8.2. INFINITE SERIES 453 ∞1 and the finite sum a1 +· · ·+am−1 does not 37. Let Sn = . Then affect convergence, both series converge or k=1 k both diverge. 1 3 5 S1 = 1 and S2 = 1 + = . Since S8 > , we 2 2 2 34. By using partial fraction technique, we get: have 1 1 1 1 1 1 S16 = S8 + + + ··· + 1 =2 − 1 9 10 16 k k+ 2 k k+ 2 1 1 5 1 It may be observed that unlike example 2.3 > S8 + 8 = S8 + > + = 3. 16 2 2 2 1 1 So S16 > 3. both and do not form the same se- k k+ 1 1 1 1 2 S32 = S16 + + + ··· + ries. Consequently, the terms in the nth par- 17 18 32 tial sum do not get canceled. Hence the said 1 1 1 7 > S16 + 16 = S16 + > 3 + = method is not useful for this example. 32 2 2 2 7 ∞ ∞ So S32 > . 2 35. Assume ak converges to A and bk con- If n = 64, then S64 > 4. If n = 256, then k=1 k=1 S256 > 5. If n = 4m−1 , then Sn > m. verges to B. Then the sequences of partial sums converge, and letting 38. The nth partial sum of the series n n Sn = ak and Tn = bk , P = 1 − 1 + 1 − 1 + 1 · ·· is given by n k=1 k=1 k 1 n we have lim Sn = A and lim Tn = B. Pn = (−1) = (1 + (−1) ). n n 2 k=0 n Here Sn = the nth partial sum of the series Let Qn = (ak + bk ), the sequence of partial 1 − 1 + 1 − 1 + 1 · ·· k=1 ∞ 1 n Which is =Sn = 1+ (1 + (−1) ). sums for (ak + bk ). Since S, T , and Q are 2 n k=1 k As (−1) diverges, by theorem 2.3 the se- all finite sums, k=0 Qn = Sn + Tn . ries 1 + 1 − 1 + 1 − 1 + 1 · ·· diverges. Then by Theorem 1.1(i), A + B = lim Sn + lim Tn = lim(Sn + Tn ) For the series A = 1 + 1 − 1 + 1 − 1 + 1 · ··, nth n n n n−1 ∞ k partial sum is given by, Sn = 1 + (−1) . = lim Qn = (ak + bk ) k=0 n ∞ k=1 ∞ Now, for the series The proofs for (ak − bk ) and cak are P = 1 − 1 + 1 − 1 + 1 · ·· we can write: P =1−1+1−1+1······ k=1 k=1 similar. 1 − P = 1 − (1 − 1 + 1 − 1 + 1 · · · · · ·) = P ∞ ∞ 1 36. Assume that ak converges and bk di- ⇒P = 2 k=1 k=1 ∞ For the 1 + 1 − 1 + 1 − 1 + 1 · · · · · · we have: verges. Now suppose that (ak + bk ) con- A = 1 + P = 1 − 1 + 1 − 1 + 1 · ·· k=1 verges. But, since 1 3 ∞ ∞ ∞ =1+ = 2 2 bk = (ak + bk ) − ak The Cesaro sum n k=1 k=1 k=1 1 ∞ lim Sk n→∞ n Theorem 2.3(i) implies convergence of bk . k=1 n k=1 1 3 ∞ = lim · lim Sk =0· =0 n→∞ n n→∞ 2 This contradiction shows that (ak +bk ) can- k=1 k=1 ∞ ∞ not converge. The proof for (ak − bk ) is 39. (a) .9 + .09 + .009 + · · · = .9(.1)k k=1 k=0 almost identical. which is a geometric series with a = .9
  • 12. 454 CHAPTER 8. INFINITE SERIES and |r| = .1 < 1 so it converges to This can’t be an integer since all the terms on .9 the numerator are divisible by p except for the = 1. 1 − .1 last term. 1 9 9 (b) 0.199999 = + + + ... 46. Step 0: [0, 1] 10 100 1000 Step 1: After removing middle third, set be- ∞ k 1 9 1 1 2 = + come 0, 3 ∪ 3 , 1 10 100 10 Step 2: After removing middle third from k=0 9 1 each subinterval, set become 0, 9 ∪ 2 , 3 ∪ 9 9 1 2 6 7 8 = + 100 = 9 , 9 ∪ 9, 1 10 1 − 1/10 10 18 18 All the end points of the subintervals belong 40. (a) 0.181818 = + + ... to Cantor Set. Therefore other four points are 100 10000 ∞ 1 2 7 8 1 1 9 , 9 , 9 , 9 . Third term of the series is 4 27 . = 18 . Sum of the series: 100k k=1 1 1 1 This is a geometric series with = +2 +4 + ... 18 1 3 9 27 a = and |r| = , so the sum is 1 1 1 100 100 = +2 + 22 + ... 18/100 2 3 32 33 = . 1 − 1/100 11 1 1 1 134 134 = 1+2 + 22 + ... (b) 2.134134 = 2 + + + ... 3 3 32 1000 1000000 1 1 ∞ 1 = 2 = 2 + 134 . 3 1− 3 1000k k=1 =1 The second term is a geometric series with 134 1 a= and |r| = , so the sum is The length of the Cantor Set 1000 1000 = 1 − (Sum of the lengths of remove intervals) 134/1000 134 2+ =2 . = 1 − (Sum of the series) 1 − 1/1000 999 ∞ ∞ =1−1 1 −1 =0 41. and k k k=1 k=1 47. We have from the proof of the Theorem 2.1, 42. Yes. 1 1 − xn+1 1+x+x2 +x3 +x4 +........+xn = . 1−x 43. The geometric series 1 + r + r2 + r3 + · · · con- Now consider 1 0 < x < 1 ⇒ 0 < xn < 1 verges to S = provided that 1−r − 1 < −xn < 0 or 0 < 1 − xn < 1 −1 < r < 1. But −1 < r implies that −r < 1, 1 1 1 − xn 1 and so 1 − r < 2, and therefore > , therefore 0 < < 1−r 2 1−x 1−x 1 1 which means that S > . ⇒ 0 < 1 + x + x2 + ... + xn < 2 1−x 44. Since ak converges, lim ak = 0, Now, for the interval −1 < x < 0 k→∞ 1 0 < −x < 1 by Theorem 2.2. Therefore, lim does not 1<1−x<2 k→∞ ak 1 If n is even, −1 < xn+1 < 0. Then exist. In particular, lim = 0. 0 < −xn+1 < 1 k→∞ ak 1 ⇒ 1 < 1 − xn+1 < 2 Therefore, diverges, by the converse of 1 1 − xn+1 2 ak ⇒ < < Theorem 2.2. 1−x 1−x 1−x 1 − xn+1 45. Suppose p is a prime, Therefore, 1 + x + x2 + ... + xn = > 1−x 1 1 1 p! + p!/2 + · + p!/p 1 1+ + +·+ = 2 3 p p! 1−x
  • 13. 8.2. INFINITE SERIES 455 Thus, the inequality does not hold if n is even. uct on the left side of the inequality. This product will yield the sum of the terms of the 1 ∞ form ,where k is either a prime or a product 1 k 48. The series is a geometric series with of the primes or the product of a prime and a 2k composite or it is the product of two compos- k=0 1 ites, so that we can write the left side of the a = 1 and r = , its sum is 2. 2 1 1 1 1 ∞ inequality as 1 + + + + · · · · + ,where 1 2 3 4 n Therefore, = 2. n is a positive integer greater than the largest 2k k=0 prime p. Now, the nth partial sum of this series is given Thus, we have by 1 1 1 1 3 5 p n 1 1 n 1+ + + +····+ < 2· · ·········· =2 1− < 2 2 3 4 n 2 4 p−1 2k 2 It may be observed that, if there would have k=0 1 1 been finite number of primes then there would Therefore 1 + + ........ + n < 2 ...(i) have been finite number of terms on either side 2 2 ∞ of the inequality, which is not the case hence 1 The series is a geometric series with we conclude that there are infinite number of 3k k=0 primes. 1 3 a = 1 and r = , its sum is . n−1 3 2 L ∞ 49. The amount of overhang is . 1 3 2(n − k) Therefore, = k=0 3k 2 7 k=0 L th So if n = 8, then = 1.3589L. Now, the n partial sum of this series is given 2(8 − k) by k=0 n 1 3 1 n 3 When n = 4, = 1− < n−1 3 3k 2 3 2 1 1 k=0 L =L 1 1 3 2(n − k) 2(4 − k) k=0 k=0 Therefore, 1 + + ........ + n < ...(ii) 3 3 2 = 1.0417L > L ∞ 1 n−1 n The series is a geometric series with L L 5k lim = lim k=0 n→∞ 2(n − k) n→∞ 2k 1 5 k=0 k=1 a = 1 and r = , its sum is . n 5 4 L 1 ∞ = lim =∞ 1 5 2 n→∞ k Therefore, = k=1 5k 4 k=0 n n th Now the n partial sum of this series is given 1 1 1 50. = − by k(k − 1) k−1 k k=2 k=2 n n 1 4 1 5 1 1 1 1 1 1 = 1− < = − + − + ··· + − 5k 5 5 4 1 2 2 3 n−1 n k=0 1 1 1 5 = 1 − , we have Therefore, 1 + + ........ + n < ...(iii) n 5 5 4 ∞ In general we can write for a largest prime p, 1 1 = lim 1 − = 1. 1 1 p k(k − 1) n→∞ n 1 + + ........ + n < k=2 p p p−1 On the other hand, Now, multiplying the inequalities ∞ ∞ 1 1 (i),(ii),(iii).etc. We get, k = k(k − 1) (k − 1) 1 1 1 1 k=2 k=2 1+ +····+ n 1+ +····+ n ∞ 2 2 3 3 1 = = ∞. 1 1 1 1 k 1+ +····+ n ··· 1+ +····+ n k=1 5 5 p p 2 3 5 p 51. p2 + 2p(1 − p)p2 + [2p(1 − p)] p2 + · · · ∞ < 2· · ·········· k 2 4 p−1 = p2 [2p(1 − p)] is a geometric series with for any positive integer n. Consider the prod- k=0
  • 14. 456 CHAPTER 8. INFINITE SERIES a = p2 and |r| = 2p(1−p) < 1 because 2p(1−p) The sum represents the probability that you is a probability and therefore must be between eventually win a game. 0 and 1. So the series converges to p2 p2 = . 1 − [2p(1 − p)| 1 − 2p + 2p2 If, p = .6, 2v ∞ .62 2v g 2v 1 − 2(.6) + 2(.6)2 = .692 > .6. 58. rk = = g 1−r g(1 − r) k=0 1 p2 2v If p > , > p. ∞ 2 1 − 2p(1 − p) 2v g 2v r2k = = 1 1 1 ∞ k g 1 − r2 g(1 − r2 ) k=0 52. Since 1 + + 2 + ··· = 12 12 12 k=0 1 12 = = , the minute hand and the hour 1 11 1− 59. (a) Take a number less than 1000 and try. 12 hand are in the same location approximately 5 minutes and 27 seconds after 1. (b) The series ∞ c c c 53. d + de−r + de−2r + · · · = d(e−r )k c + 6 + 12 + 18 + .......... is an in- 10 10 10 k=0 finite geometric series with a = c and which is a geometric series with 1 a = d and e−r = e−r < 1 if r > 0. r= 6 <1, 10 ∞ c k d it converges to So d(e−r) = . 1 − 10−6 1 − e−r k=0 where If r = .1, x 999−x c 103 + 106 −6 = ∞ d 1 − 10 1 − 10−6 d(e−.1 )k = =2 1 − e−.1 1000x + 999 − x 999x + 999 k=0 so d = 2(1 − .905) ≈ .19 = = 999999 999999 ∞ k x+1 15 1 = 54. 30 + 15 + + ··· = 30 1001 2 2 k=0 30 = = 60 miles. (c) Let x + 1 be a positive inte- 1 − 1/2 ger(assumed) less than 1000 such that The bikes meet after 1 hour. In that time, a x+1 fly flying 60 mph will have traveled 60 miles. /11 /13 = 7 ∞ k 3 100,000 x+1 55. 100,000 = = $400,000. = 4 1 − 3/4 1001 k=0 c c c = c + 6 + 12 + 18 + ..... 56. P = ce−r + ce−2r + ce−3r + . . . 10 10 10 ∞ ∞ k x 999 − x x 999 − x = ce−kr = c e−r = 3+ 6 + 9+ 10 10 10 1012 k=1 k=1 x 999 − x ce−r c + 15 + +··· = = r 10 1018 1 − e−r e −1 First three digits are given by the term x 57. Since 0 < p < 1, therefore −1 < −p < 0, and only. Therefore the first three dig- 0 < 1 − p < 1. Thus, the given series is geo- 103 x+1 metric with common ratio r = 1 − p, and so its of are the digits of x. Thus 103 converges: here the digits of the number on the left ∞ ∞ side repeat after every 6 digits and first p(1 − p)n−1 = p rn−1 three digits ( i.e x) is less by one than the n=1 n=1 1 p p assumed number (x + 1) and last three =p = = = 1. digits are 9’s complements of x. 1−r 1 − (1 − p) p